切换到宽版
  • 3292阅读
  • 16回复

[资料]生理英文题 [复制链接]

上一主题 下一主题
离线孙笑天
 
发帖
97
啄木币
1729
鲜花
82
只看楼主 倒序阅读 使用道具 0楼 发表于: 2007-12-15
— 本帖被 sduqingfeng 从 VIP资料区 移动到本区(2007-12-16) —
英文题  第六章(06. 3.17)
Ⅰ. choise question
1. Which of the following descriptions about the general characteristics of smooth muscle of alimentary tract is wrong?  E
  A. Its excitability of the smooth muscle of the gut is low.
B. It has autorhythmicity.
C. It has tonic contraction.
D. It has high extensibility.
E. It is not sensitive to stretch, chemical, cold, and warm stimulation.
2. Which of the following descriptions about the tonic contraction of smooth muscle of alimentary tract is wrong?  E
A. It is a common movement type of stomach and small intestine.
B. To keeps the stomach and the intestine in their normal shape and location.
C. To maintains a given pressure in alimentary tract lumen
D. To provides an adequate background for other forms of movement
E. When tonic contraction become weak, the absorption of gastrointestinal tract increases.
3. The rhythm of the spontaneous contraction of gastrointestinal smooth muscle is determined mainly by  A
A. the rhythm of the slow wave.
B. the level of the resting membrane potential.
C. the frequency of the action potential.
D. the amplitude of the action potential.
E. the level of activity of the sympathetic nerve.
4. Which of the following descriptions about the slow wave of smooth muscle of alimentary tract is wrong? E
A. It is the spontaneous depolarization and repolarrization
B. It originate in the Cajal cell.
C. After the nerves innervating the gut are cut, the slow wave still persist.
D. It may be the pacemaker potential for smooth muscle.
E. It cause directly the smooth muscle to contrat.
5. Functional base of the autorhythmicity of the smooth muscle of gastrointestinal tract is  C
A. The action potential. 
B. The resting membrane potential
C. The basic electrical rhythm (BER).
D. The tonic contraction.
E. the smooth muscle spontaneous contraction.
6. The cause initiating slow wave is  A
A. A slow undulation of the activity of the Na+-K+ pump.
B. A spontaneous depolarization and repolarization of cell membrane
C. Na+-Ca2+ exchange.
D. I k progressively attenuate decrement
E. Ca2+ inflow
7. Ionic base of the rising phase of action potential in smooth muscle of gastrointestinal tract is  A
A. inflow of Ca2+ and small amount of
B. Na+ inflow
C. Ca2+ inflow
B. K+ outflow
D. Cl- outflow
E. Cl-  inflow
8. Which of the following descriptions about the innervation of digestive organs is correct ?  C
A. Postganglionic fiber terminals of sympathetic nerve release acetylcholine (Ach)
B. All postganglionic fiber terminals of parasympathetic nerve release Ach
C. After the external innervation of the gut are cut, the short reflexes may be completed.
D. Activity of sympathetic nerve increases the movement of gastrointestinal tract.
E. Activity of parasympathetic nerve inhibits the secretion of digestive glands.
9. Which of the following descriptions about enteric nervous system is wrong ?  E
A. The number of neurons in this intrinsic nervous system is almost equal to the number in spinal cord.
B. It contains sensory neurons, motor neurons and intermediate neurons.
C. It can complete a short reflex without the external innervation. 
D. It regulates the movement and glands secretion of gastrointestinal tract. 
E. It is not affected by the external innervation.
10. Which of the following is not the function of HCl ?  E
A. To activate the pepsinogen
B. To kill bacteria
C. To stimulate the upper small intestinal mucosa to  release secretin
D. To promote absorption of the ferrum and calcium in the upper small intestinal.
E. To promotes vitamin B12 to be absorbed
11. Which of the following descriptions about the secretion of the gastric acid is wrong?  E
A. It is secreted by the parietal cells.
B. The intracellular H+ is transported by proton pump into the canaliculus lumen.
C. The omeprazole (奥美拉唑) inhibits strongly the secretion of gastric acid.
D. When one H+ is secreted, one HCO3- enters blood.
E. HCl in gastric lumen do not affect the gastric secretion. 
12. The location where the vitamin B12 to be absorbed is  C
A. Stomach
B. Duodenum
C. Terminal ileum
D. Jejunum
E. Large intestine
13. The substance that activates the pepsinogen into pepsin is  B
A. Enterokinase
B. HCl and pepsin
C. Histamine
D. Trypsin
E. Chymotryosin
14. Which of the following is not the function of gastrin  D
A. To promote the parietal cell to secrete HCl
B. To promote the pancreatic acini to secrete digestive enzyme
C. To promote the liver to secrete the bile
D. To promote the absorption of fat acid
E. To have the trophic action to gastrointestinal mucosa 
15. Which of the following inhibits the G cells from releasing gastrin  B
A. The intragastric food stretches gastric wall. 
B. The pH in the gastric antrum is below 1.5
C. The intragastrioc peptones and the proteoses and peptides increase
D. The activity of vagus nerve increases
E. The short reflex caused by stretch of food to the wall of gastric antrum
16. The transport manner by which the parietal cells secrete H+ is  A
A. Primary active transport
B. Second active transport
C. Facilitated diffusion
D. Simple diffusion
E. Exocytosis
17. After blocking the M receptors by the atropine, the change of the gastrointestinal tract is
A. The secretion of digestive glands increases.  E
B. The slow waves lose
C. The tonic contraction loses
D. The peristalsis loses 
E. The tonic contraction and peristalsis weaken 
18. Which of following substances do not increase the gastric secretion  E
A. Gastrin
B. Histamine
C. acetylcholine
D. Activity of vagus nerve
E. Somatostatin
19. Which of following substance do not inhibit gastric secretion  E
A. Somatostatin
B. Secretin
C. Prostaglandins
D. H2 receptor antagonist,
E. Protein foods
20. Which of the following descriptions about the cephalic phase of gastric secretion is wrong?  A
A. It is pure nervous regulation.
B. It involves conditioned reflex and unconditioned reflex. 
C. M receptors antagonists decrease gastric secretion in this phase.
D. It can eliminated by cutting bilateral vagus nerves
E. The secretion has high acidity, contains a copious amount of pepsin.
21. Which of the following descriptions about the gastric phase of gastric secretion is wrong? 
A. It is initiated mainly by the distention of food on gastric wall.
B. It involves the vagus-vagus long reflex
C. It It involves the short reflex of enteric nervous system.
D. It do not involves the release and function of the gastrin
E. The secretion has a low acidity, but its pepsin content is higher than that in the cephalic phase 
22. One of mechanism by which HCl inhibit is  C
A. Intragastric HCl promotes G cells to release gastrin.
B. Intragastric HCl inhibits D cells from releasing somatostatin.
C. Intraintestinal HCl causes S cells secrete scretin 
D. Intraintestinal HCl inhibits duodenal mucosa from secreting bulbogastrone.
E. HCl inhibits directly the parietal cells.
23. Movement type peculiar to stomach is      B
A. Tonic contraction
B. Receptive relaxation
C. Migrating motor complex
D. Peristalsis
E. Mass movement
24. After cutting the bilateral vagus nerves, the gastric receptive relaxation will  A
A. Lose
B. Weaken
C. Strengthen 
D. No change
E. Weaken or strengthen
25. Which of the following descriptions about the gastric peristalsis is wrong  E
A. It is a cooperating movement of the longitudinal muscle and circular muscle
B. It is one of the motivity of gastric empting
C. It begin in the mid portion of the body of the stomach.
D. It is about 3 times per minute.
E. It does not partake in mixing, and grinding the stomach contents.
26. Which of the following descriptions about the migrating motor complex is wrong  A
A. It is movement type peculiar to small intestine.
B. It occurs on fasting period
C. It propagates from the stomach to the terminal ileum
D. In humans, the MMC repeats every 75 to 90 minutes
E. It inhibits the migration of colonic bacteria into the small terminal ileum
27. Which of the following inhibits the gastric empting  D
A. The long vagus-vagus reflex
B. The locally short reflex
C. Acetylcholine
D. The enterogastric reflex
E. Histamine
28. Which of the following promotes the gastric empting  A
A. The increased intragastric food volume
B. The stimulation of HCl to the duodenal mucosa
C. The increased fatty content of the chyme in duodenum
D. The increased osmotic pressure of the chyme in duodenum
E. Secretin and gastric inhibitory peptide
29. Which of the following descriptions about the gastric empting is wrong  E
A. Five minutes after the food enters stomach, the stomach begins to empty.
B. Its direct driving force is the pressure difference between the stomach and the duodenum.
C. Its motivity is the peristalsis and gastric tonic contraction
D. The gastric emptying rate of fat is the lowest. that of saccharide is the fastest,
E. Acetylcholine inhibits gastric empting
30. The most important digestive juice in human is  B
A. Gastric juice
B. Pancreatic juice
C. Bile
D. Small intestinal juice
E. Large intestinal juice
31. The most potent stimulus for secretin release from S cells in upper small intestinal mucosa A
A. HCl
B. Protein digestive products
C. Fatty acid
D. Saccharides
E. Efferent impulses of vagus nerve
32. The most potent stimulus for CCK release from D cells in upper small intestinal mucosa  B
A. HCl
B. Protein digestive products
C. Saccharides
D. Saccharides
E. Efferent impulses of vagus nerve
33. Factor causing the pancreas to secrete a large amount of bicarbonate and water is  A
A. Secretin
B. Gastrin
C. Acetylcholine
D. CCK
E. Histamine
34. The most important factor resulting in secretion of pancreatic enzymes is  B
A. Somatostatin
B. CCK
C. Secretin
D. Gastrin
E. Activity of vagus nerves
35. Type of movement peculiar to the small intestine is  E
A. Tonic contraction
B. Receptive relaxation
C. Migrating motor complex
D. Peristalsis
E. Segmentation movement
38. Main mechanism by which the acidic chyme in the duodenum causes pancreases to secrete large amount of bicarbonate and water is  A
A. Small intestinal mucosa releases secretin
B. Small intestinal mucosa releases CCK
C. Small intestinal mucosa releases gastrin
D. Vagus-vagus reflex - gastropancreatic reflex
E. Antral- pancreatic reflex
37. Main mechanism by which the proteoses and peptones in the duodenum causes pancreases to secrete large amount of pancreatic enzyme is  B
A. Small intestinal mucosa releases secretin
B. Small intestinal mucosa releases CCK
C. Small intestinal mucosa releases gastrin
D. Activity of vagus nerves increases
E. Activity of sympathetic nerves increases
38. Characteristic of pancreatic secretion caused by the activity of vagus nerves and gastrin is  A
A. Small volume but rich in enzymes.
B. Large volume and rich in enzymes
C. Large volume but poor in enzymes
D. Small volume and poor in enzymes
E. It can not be blocked by atropine 
39. Which of the following descriptions about segmentation movement of small intestine the is wrong  E
A. It is caused by the distention of chyme to small intestinal wall.
B. It move the intestinal content forth and back over short distance
C. It promote absorption of the digestive products
D. It promote the return of blood and lymph
E. It does not relate to the enteric nervous system
40. Which of the following descriptions about the function of bile is wrong  E
A. There is no any digestive enzyme in the bile.
B. Function of bile in digestion of food is accomplished mainly by the bile salts.
C. Cholates promote the absorption of liposoluble vitamin。
D. Cholates help in the absorption of fatty acid.
E. Cholates promote cholesterol deposition.
Ⅱ.Define following word
1.Digestion  Digestion refer to process by which the different foods in the alimentary tract are resolved into small molecular components.
2.Mechanical digestion. The contractile activity of muscle of alimentary tract grinds food, mixes it with the digestive juice, and propels the luminal contents from proximal to distal alimentary tract, which is called the mechanical digestion..
3.Chemical digestion The process by which various special enzymes contained in the digestive juice breakdown foodstuffs into simple and absorbable forms
4.Absorption The process that the digested foods, water, electrolytes, and vitamins are transported across the gastrointestinal tract mucosa into blood and lymph is called the absorption.
5.Basic electric rhythm (BER) Usually, the smooth muscle of alimentary tract depolarizes and repolarizes spontaneously and slowly in a cyclic fashion; this electric activity is called slow wave or basic electrical rhythm (BER).
6.Gastrointestinal hormone or gut hormone The various hormones secreted by the endocrine cells in the gastrointestinal mucosa are collectively called the gastrointestinal hormone or gut hormone. All the gastrointestinal hormones are the peptide. 
7.Brain-gut peptide: Many peptide called the gastrointestinal hormones are also the transmitter of neurons in certain regions of central nervous system; these double distributing peptides are collectively called the brain-gut peptide.
8.Mucus – bicarbonate barrier The layer of insoluble mucus and bicarbonate constructs a barrier protecting the stomach mucosa from injury by the gastric juice and food, which is called mucus-bicarbonate barrier.
9.Cephalic phase of gastric secretion  When food is being eaten, the sensory receptors activating the reflex of gastric secretion are located on the head, this is called cephalic phase of gastric secretion.
10.Gastric phase of gastric secretion  After entering the stomach, food stimulates the mechanical and chemical receptors in the stomach wall, thus causing continuously gastric secretion, which is called gastric phase of gastric secretion.
11. Receptive relaxation  During chewing and swallowing food, the stimulation of food to the receptors in mouth, pharynx, and esophagus causes reflexly the smooth muscle in the fundus and body of stomach to relax, which is called the receptive relaxation.
12. Peristalsis  Peristalsis is wave-like propulsive movement carried out by a cooperating movement of the longitudinal muscle and circular muscle in the alimentary tract.
13. Migrating motor complexm  During fasting (or in interdigestive period, or in nondigestive period ), the stomach and small intestine exhibit a periodical movement that is characterized by strong propulsive contraction and having longer quiescent period, which is called migrating motor complex (MMC).
14. Enterogastric reflex  After entering the duodenum, HCl, fat, high osmotic pressure, and the distention of the chyme stimulate a variety of receptors in the duodenum and thus reflexly inhibits gastric motility, gastric secretion, and gastric emptying, which is called enterogastric reflex.
15.Entero-hepatic circulation of bile salts  About 90 to 94 per cent of bile salts emptied into duodenum are reabsorbed by the terminal ileum, are then returned by portal vein to liver, where they are removed from blood and re-secreted into bile; this process is called entero-hepatic circulation of bile salts
16.Segmentation movement  When a portion of the small intestine becomes distended with chyme, the stretch of the intestinal wall elicits a rhythmical contraction and relaxation of localized circular muscles spaced at intervals along the intestine, which is called segmentation contraction or segmentation movement.
17. Peristaltic rush  The intense irritation of the small intestinal mucosa can cause both powerful and rapid peristalsis, called the peristaltic rush.
18.Mass movement  Mass movement is a colonic peristalsis propelling rapidly long distance.

Ⅲ.Answer following question
1. Describe the composition, and function of gastric juice
(1) Hydrochloric acid (gastric acid)
① HCl in stomach activates the pepsinogen into pepsin and provide a favorable acidic environment for the optimal activity of this enzyme.
② It kills bacteria that enter the digestive tract with food.
③ After entering the small intestine, it stimulate the upper small intestinal mucosa to release secretin and cholecystokinin, which in turn cause pancreatic juice, pancreatic enzyme, and bile to be secreted.
④ It lowers pH in small intestine, thus promoting absorption of the ferrum and calcium in this region.
(2) Pepsinogen
The pepsin hydrolyzes protein into the peptones, the proteoses, and small quantities of little peptide and amino acids.
(3) Intrinsic factor
Intrinsic factor promotes the absorption of vitamin B12 in the lower small intestine (ileum).
(4) Mucus and bicarbonate
They help to lubricate food movement as well as to protect the stomach wall from mechanical trauma.
2. Describe mechanism of hydrochloric acid secretion
The apical membrane (luminal membrane) of the parietal cell dents inward and thus forms secretory canaliculus (分泌小管). There are proton pumps and Cl- channels on the membrane of secretory canaliculus.
  In parietal cell            Carbonic
anhydrase
              CO2 + H2O ←———→ H2CO3 → H+ + HCO3-     
The intracellular H+ is actively transported by the proton pump against its electrochemical gradient into the canaliculus lumen and then into acinar lumen. The intracellular HCO3- is transported by Cl-–HCO3- antipoter located on the basolateral membrane of the parietal cells to interstitial fluid and then into blood; in the same time, the Cl- enter the cell,  then diffuse via Cl- channel into the canaliculus lumen and then into. In acinar lumen, Cl- and H+ form HCl.   
The proton pump inhibitor, such as omeprazole, decrease the secretion of gastric acid.
3. Describe endogenous substances stimulating gastric secretion 
(1)Acetylcholine: acetylcholine (Ach) is the nervous transmitter of most of vagus nerves and part of enteric nervous system.
Function:
1). Ach binds with the muscarinic receptor (M receptor) on the gastric glandular cell membrane, and thus cause directly gastric gland to secret gastric acid and pepsinogen.
2). Ach binds with the M receptor on cellar membrane of G cells and ECL cells in the gastric mucosa to cause G cells to release gastrin and ECL cells to release histamine, indirectly increasing secretion of gastric juice.
3). Ach inhibits D cells from releasing somatostatin and thus increases indirectly the secretion of gastric juice.
The M receptor blocker is atropin
(2)Gastrin: Gastrin is a peptide secreted by the G cells of the antral mucosa and duodenal mucosa.
The nervous signals from the vagus nerve,the local short reflexes, and protein digestive products cause the G cells to secrete gastrin.
Gastrin and is carried by blood to the oxyntic glands where it stimulates the parietal cells to secrete HCl and promotes slightly the chief cells to secrete pepsinogen.
The antagonist of gastrin receptor is proglumide.
(3) Histamine: The histamine secreted by the enterochromaffin-like cells (ECL cells) diffuses via local interstitial fluid to neighbor parietal cells, binds with the H2 recepors on the parietal cells and thus causes parietal cells to secrete HCl.
The H2 receptor antagonist, such as cimetidine and ranitidine, decreases effectively HCl secretion and thus are used to cure (treat) the gastric ulcer.
There are the receptors of Ach and gastrin on the enteromaffin-like cells. Both Ach and gastrin promote the ECL cell to secrete histamine.
Ach, gastrin and histamine act synergistically. All three of receptors to these separate transmitter-hormonal substances must be activated simultaneously to give a truly effective stimulus for HCl secretion.
4. Describe endogeneous substances inhibiting gastric secretion
(1) Somatostatin (SS): Somatostatin is secreted by the D cells in the gastric antral and fundus mucosa and intestinal mucosa.
    Somatostatin strongly inhibit the parietal cells from secreting HCl. Somatostatin can directly inhibit parietal cell to secrete gastric acid; inhibit the G cells to secrete gastrin, inhibits the ECL cells to secrete histamine.
(2) Prostaglandins (PGs): Gastric mocusa secrete PGs. PGs powerfully inhibit gastric acid secretion. Drugs inhibiting PG synthesis, such as indomethacin and aspirin, increase significantly gastric acid secretion and cause gastrohelcosis (or gastric ulcer).
(3)Secretin: Secretin is secreted by the S cells in the mucosa of upper small intestine. Sectetin inhibit gastric secretion.
(4) 5-hydroxytryptamine (5-HT): 5-TH is the neurous transmitter of the enteric plexus. It inhibits the gastric secretion caused by the gastrin.
5. Describe mechanism and characteristic of cephalic phase of gastric secretion 
The conditioned reflex:  The sight, smell, and sound of food stimulate eyes, nose, ears; signals are transmitted via the corresponding afferent nerves into central nervous system; gastric secretion increases reflexly.
It requires acquired experiences.
The unconditioned reflex: Food stimulates the mechanical and chemical receptors in mouth, pharynx, and esophagus; signals are transmitted via the corresponding afferent nerves into central nervous system; gastric secretion increases reflexly.
It do not require acquired experiences.     
The reflex centers of the cephalic phase of gastric secretion include cerebral cortex, limbic system, and medulla.
The common efferent nerve of the conditioned and unconditioned reflexes is vagus nerve. Vagal efferent impulses ① cause directly gastric glands to secrete gastric juice (Ach-M receptor), ② promote via a intermediate neuron the G cells in gastric antral mucosa to release gastrin  ③stimulate ECL cells to release histamine (Ach-M receptor), the gastrin and histamine in turn causing gastric secretion. Therefore, the cephalic phase of gastric secretion is a nervous-humoral regulation. .
Characteristics: ① the secretion has high acidity, contains a copious amount of pepsin. ② the better the appetite, the more the secretion of gastric juice. ③ This phase of secretion accounts for about 30% of the gastric secretion associated with eating a meal.
6.Describe mechanism and characteristic of gastric phase of gastric secretion
Mechanism: The food in stomach triggers this phase of gastric secretion via three pathway:
①The distention of food on the fundus and body of stomach activates a vagus-vagus long reflex and short reflex of enteric nervous system. These reflexes cause directly and indirectly gastric secretion.
② The food stretches the wall of pyloric antrum, thus give rise to a local short reflex to stimulate the G cells to release gastrin.
③The protein digestion products directly stimulates the G cells to release gastein.
Characteristics: ① In this phase, the secreted gastric juice has higher acidity but pepsin content is lower than that in the cephalic phase.  ② The gastric phase of secretion accounts for the about 60% of the total gastric secretion associated with eating a meal.
  7. Describe factors inhibiting gastric secretion during during digesting food
(1) Hydrochloric acid (HCl)
HCl secreted by gastric glands can inhibit gastric secrettion. This is a negative feedback. HCl decreases gastric secretion by two pathway. ① Intragastric pathway: As pH in pyloric antral region falls below 1.5 to 1.2, gastric secretion is inhibited significantly, partly because this acidity suppresses directly the release of gastrin, partly because the intragastric HCl cause the D cells of gastric mucosa to release somatostatin, which in turn inhibits the gastric secretion and release of gastrin. ② Intraintestinal pathway: Intraintestinal HCl cause duodenal mucosa to release secretin and bulbogastrone, both hormones inhibiting gastric secretion.
(2) fat
Fat inhibits gastric secretion via small intestinal pathway. The presence of fat in upper small intestine causes the intestinal mucosa to release enterogastrone , the latter inhibits gastric secretion. However, the enterogastrone is postulated to be actually secretin, gastric inhibitory peptide.
(3) Hypertonic solution
The presence of hypertonic solution in the upper small intestine decreases the gastric secretion by two pathway. ① The hypertonic solution stimulates the osmotic pressure receptor in small intestinal wall and thus initiates an enterogastric reflex, which inhibits gastric secretion. ② The hypertonic solution causes the intestinal mucosa to release several hormones to inhibit gastric secretion.
8. Describe the control of gastric emptying
(1) Intragastric factors promoting gastric emptying
The rate of gastric emptying is directly proportional to the square root of intragastric food volume. ① The distention of food on the gastric wall activates the long vagus-vagus reflex and the locally short reflex. These reflexes enhance directly the gastric tonic contraction and peristalsis via Ach and M receptor. ② The short reflex caused by stretch of food to the wall of gastric antrum and the protein digestion products cause the G cells to release gastrin, the gastrin in turn promotes gastric movement.
2) Intraduodenal factor inhibiting gastric emptying
After entering the duodenum, the chyme inhibits gastric emptying by two pathway.
① The enterogastric reflex: After entering the duodenum, HCl, fat, high osmotic pressure, and the distention of the chyme stimulate a variety of receptors in the duodenum and thus reflexly inhibits gastric motility, gastric secretion, and gastric emptying, which is called enterogastric reflex.
The enterogastric reflex can be transmitted through the enteric nervous system as well as the external sympathetic and vagus to the stomach.
② Hormonal feedback from the duodenum:
After entering the duodenum, the hydrochloric acid and fat in the chyme stimulate the upper small intestinal mucosa to release several hormones, such as secretin, gastric inhibitory peptide, cholecystokinin, and so on. These hormones inhibit gastric motility and emptying.
9. Describe the composition and function of pancreatic juice
(1) Aqueous juice
Its bicarbonate concentration is high relative to that of plasma. When the pancreas is stimulated to secrete copious quantities of pancreatic juice, the bicarbonate ion concentration can rise to as high as 140 mmol/liter. The anions of pancreatic juice are mainly HCO3-and Cl- . Its cations are Na+ and K+, and these are present in the same concentration as in plasma at all rate of secretion.
The function of bicarbonate is to neutralize gastric acid emptied into the duodenum and provide a favorable alkaline environment for optimal activity of the pancreatic enzymes.
(2) Pancreatic enzymes
1) Pancreatic amylase: The pancreatic amylase hydrolyzes starches, glycogen, and most other carbohydrates to form maltose and other disaccharides and a few trisaccharides; its optimal pH is 6.7 to 7.0.
2) Pancreatic lipase: The pancreatic lipase is capable of hydrolyzing neutral fat into fatty acids and monoglycerides.
3) Trypsin, chymotrypsin, and carboxypolypeptidase
The trypsin and chymotryosin are important proteolytic enzymes. The combined action of both trypsin and chymotrypsin can degrade protein into small peptides and amino acids. The carboxypolypeptidase breaks down the peptides into amino acids.
Before entering the small intestinal lumen, the proteolytic enzymes are in inactive form of trypsinogen, chymotrypsinogen, and procarboxypolypeptidase, which are all enzymatically inactive. In addition, the pancreatic acini secret a substance called trypsin inhibitor to prevent the trypsinogen from being activated. Therefore the autodigestion of pancreas do not occur. 
After entering the intestinal tract, the trypsinogen is activated by the enterokinase in small intestinal juice into trypsin. Also, trypsinogen can be autocatalytically activated by trypsin. The chymotrypsinogen is activated by trypsin into chymotryosin. In addition, trypsinogen is also activated by the HCl and interstitial fluid.
10. Describe the regulation of pancreatic secretion
During fasting, the pancreas is at rest or secrete merely a few pancreatic juice.
Pancreatic secretion during digestive period includes three phases.
(1)Cephalic phase of pancreatic secretion
The stimulation of food to the receptors in eyes, nose, ears, mouth, pharynx, esophagus causes the conditioned and unconditioned reflexes. These reflexs cause pancreatic secretion of small volume but rich in enzymes.
The efferent nerve of these reflexes is vagus nerve. The activity of vagus nerve causes directly pancreatic glands to secrete gastric juice via Ach- M. In addition, the activity of vagal nerve causes the G cells in gastric antral mucosa to release gastrin, the gastrin in turn causing pancreatic  secretion.
Pancreatic secretion in cephalic phase accounts for 25% of total pancreatic secretion after a meal.
(2)Gastric phase of pancreatic secretion
The food in stomach triggers this phase of pancreatic secretion via two pathway: ①The distention of food on the fundus and body of stomach activates vagus-vagus reflex- gastropancreatic reflex that directly cause pancreatic secretion through the vagus nerve-Ach-M receptor. In addition, the distention of food on gastric antrum initiates antral- pancreatic reflex that causes pancreatic secretion via vagal nerve Ach-M receptor. Both cutting vagal nerve and administering atropine reduce pancreatic secretion. ② The protein digestion products directly stimulates the G cells to release gastrin, the gastrin in turn causing pancreatic secretion.
Pancreatic secretion in gastric phase accounts for 5~10 % of total pancreatic secretion after a meal
(3)Intestinal phase of pancreatic  secretion 
The intestinal phase of gastric secretion is the most important regulation.
① Secetin:
Secretion of secretin: Secretin is released by S cells in upper small intestinal mucosa. The most potent stimulus for secretin release is HCl; the protein digestive products and fatty acid cause a moderate quantities of secretin to be released; vagal impulses and saccharides have not effect.
Function: Secretin causes pancreatic ductules to secrete large volume of aqueous juice that contains a high concentration of bicarbonate ion but few enzymes. Ach and CCK augment significantly the effect of secretin in stimulating secretion of water and bicarbonate.
② CCK:
Secretion of CCK: CCK is released by I cells in upper small intestinal mucosa. The most potent stimulus for CCK release is protein digestive products; HCl and fatty acid also cause its release; saccharides and vagal impulses have no effect.
Function: ① CCK binds with CCK A receptor of pancreas and thus causes pancreatic acinar cells to secrete large quantities of digestive enzymes, it accounting for 70 to 80 per cent of the total secretion of pancreatic enzymes after a meal. In addition, CCK can stimulate the vagal afferent nerve, initiating  vagus-vagus reflex, indirectly causing secretion of pancreatic enzymes. ② Another function of CCK is to stimulate strongly the contraction of gallbladder.
11. Describe function of bile
There is no any digestive enzyme in the bile.
Function of bile in digestion and absorption of food is mainly accomplished by the bile salts. Function of bile salts(cholates) includes:
(1) Cholates emulsify fat in the food, which decreases the surface tension of the fat particles and thus convert the fat globules into fat minute droplets. This increases the action area of lipase, thus promoting the digestion of fat.
(2) Cholates help in the absorption of fatty acid, cholesterol, other lipid from the intestinal tract. They do this by forming micelles with the lipids; the micelles is minute complexes; it is high soluble because of the electrical charge of the bile salts. The lipids are “ferried” in this form to the mucosa, where the lipids are then absorbed.
(3) Cholates promote the absorption of liposoluble vitamin, such as vitamin A, D, E, K,by the same way as helping in the absorption of fat digestion products.
  (4) Cholates maintain cholesterol in soluble state, preventing cholesterol from deposition, and thus protecting a person against the cholelith (gall stone).
(5) Cholates promote the secretion of bile, can be used as chologogue
12. Describe the regulation of bile secretion and emptying of gallbladder
(1) Nervous regulation
Ingesting food and the stimulation of food to gastrointestine cause the secretion of bile and gallbladder contraction through the conditioned and unconditioned reflexes. The efferent nerve of these reflexes is vagal nerve. But the effect of these reflexes are weak.
(2) Humoral regulation
1) Gastrin: Gastrin binds with the receptor on the hepatic cells and gallbladder, increasing bile secretion and enhancing gallbladder contraction. In addition, gastrin promote secretion of gastric acid; the gastric acid stimulates the intestinal mucosa to release sectretin and CCK.
2) Secretin: Secretin increase bile secretion. Secretin acts mainly on the epithelial cells of bile ductules and duct and thus causes secretion of bicarbonate-rich watery solution.
3) CCK: Intraduodenal protein digestive products, HCl, and fatty acid stimulate intestinal mucosa to release CCK. The CCK stimulates contraction of gallbladder and relaxation of Oddi’s sphincter, thus causing the emptying of gallbladder. In addition, CCK stimulates bile ductules and ducts to secret bicarbonate and water.
4) Bile salts: About 90 to 94 per cent of bile salts emptied into duodenum are reabsorbed by the terminal ileum, are then returned by portal vein to liver, where they are removed from blood and re-secreted into bile; this process is called entero-hepatic circulation of bile salts.
The bile salts are the major stimulant for secretion of bile; they act directly on hepatic cells to stimulate bile secretion in proportion to their concentration in the portal venous blood. The quantity of bile secretion per day is highly dependent on the quantity of bile salts in entero-hepatic circulation.
5) Somatostatin(SS): SS inhibits bile secretion, antagonizes the effect of CCK in contracting gallbladder and relaxing Oddi’s sphincter, promotes the formation of gall stone.

1条评分
麦田里的守望者 鲜花 +1 2007-12-15
评价一下你浏览此帖子的感受

精彩

感动

搞笑

开心

愤怒

无聊

灌水
离线孙笑天
发帖
97
啄木币
1729
鲜花
82
只看该作者 1楼 发表于: 2007-12-15
                          英文题  第八章 (2006. 3. 23)
Ⅰ. 选择题
1. The main difference between primary urine and blood plasma is:  D
  A. Glucose concentration.
B. Crystal osmotic pressure.
C. NaCl concentration.
D. Plasma protein concentration.
E. pH value.
2. Site of renin secretion is at  A
  A. Juxtaglumerular cells.
B. Extraglomerular mesangial cells.
C. Juxtaglomerular apparatus.
D. Macula densa.
E. Glomerular capillary
3. Macula densa is a  A
A. Chemical receptor.
B. Baroreceptor.
C. Stretch receptor.
D. Volume receptor.
E. Osmoreceptor.
4. Characteristic of renal blood circulation is:  A
  A. Double capillary beds.
B. Low capillary hydrostatic pressure.
C. High peritubular capillary hydrostatic pressure.
D. Smaller blood flow of renal cortex.
E. Greater blood flow of renal marrow (medulla).
5. Which of the following descriptions about urine of a normal adult is true ? D
  A. Daily urine quantity is 2 to 3 L/ day.
B. Its osmotic pressure is equal to that of plasma.
C. It contains a large amount of NaCl.
D. Urine does not contain glucose.
E. Urine contains protein.
6. Which of the followings can increase the glomerular filtration rate ? C
  A. Arterial blood pressure rises from 80 to 180 mmHg.
B. Arterial blood pressure declines from 180 to 80 mmHg.
C. A large amount of normal saline is given intravenously.
D. Excitation of sympathetic nerve.
E. Constriction of afferent arteriole.
7. The driving force for glumerular filtration is:  A
  A. Glomerular capillary hydrostatic pressure.
B. Arterial blood pressure.
C. Plasma colloid osmotic pressure.
D. Bowman’s capsular hydrostatic pressure.
E. Blood pressure of efferent arteriole.
8. The main barrier of glomerular filtration membrane is located on B
  A. Capillary endothelial cell layer.
B. Basement membrane.
C. Visceral epithelial cell (“foot cell ” or podocyte) layer.
D. Parietal epithelial cell layer.
E. Negatively charged elements of capillary endothelial cell layer.
9. If some diseases damage the negatively charged elements of glomerular membrane, which of the following will occur ? B
  A. Glomerular filtration rate increases.
B. Proteinuria (protein in urine).
C. Hematuria ( hematuresis or blood in the urine).
D. Renal blood flow increases.
E. Plasma protein concentration increases.
10. Glucose concentration of the ultrafiltrate (primary urine) is: A
  A. Similar to blood plasma.
B. Higher than that of plasma.
C. Lower than that of plasma.
D. Similar to urine.
E. Similar to tubular fluid in distal tubule.
11. Normally, which of following substances can not move through (pass across or penetrate) glomerular filtration membrane? A
  A. Plasma protein 
B. Glocose
C. Creatinin
D. urea
E. Inulin
12. When noradrenaline (norepinephrine ) is given intravenously, the main cause of urine quantity decreasing is: B
  A. Boeman’s capsule hydrostatic pressure increases.
B. Renal arteriole constricts. 
C. Release of ADH increases.
D. Release of renin decreases.
E. Release of aldosterone decrease.   
13. Which of the following descriptions about glomerular filtration membrane is wrong?  E
  A. It consists of capillary endothelium, basement membrane, and epithelial cell.
B. It is quite porous.
C. It is structural base of glomerular filtration.
D. The basement membrane is major barrier for plasma protein.
E. The negatively charged protein move easily through the filtration membrane.
14. If reabsorption of renal tubule and collecting duct to water decreases 1%, the quantity of urine will increase  D
  A. 1%
B. 2%
C. 50%
D. 100%
E. 300%
15.Which of the following descriptions about the reabsorption of Na+ and Cl- at the proximal
tubule is wrong?  E
A. The intracellular Na+ are transported by Na+ - K+ pump to interstitial fluid   
B. The process by which Na+ move from tubular fluid into cell is passive transport. 
C. In second half of proximal tubule, Na+ and Cl-diffuse from tubular lumen into intercellular fluid mainly via the tight junction.
D. The first half of proximal tubule does not reabsorb Cl- .
E. It is regulated by aldosterone
16. Proximal tubule of kidneys reabsorbs  C
  A. 85% of filtrated water.
B. 85% of filtrated Na+.
C. 65~70% of filtrated Na+ and water.
D. 65~70% of filtrated glucose.
E. 65~70% of filtrated HCO3-.
17. Location reabsorbing glucose is at: A
  A. Proximal tubule.
B. Descending limb of Henle’ loop.
C. Ascending limb of Henle’ loop.
D. Distal convoluted tubule.
E. Collecting duct.
18. Normally, proximal tubule reabsorbs:  E
  A. 20% of filtrated glucose.
B. 50% of filtrated glucose.
C. 65~70% of filtrated glucose.
D. 80% of filtrated glucose.
E. 100% of filtrated glucose.
19. The reason for that the filtration occurs only in afferent
arteriole end of the glomerular capillary is:  B
  A. Blood pressure at the efferent arteriole end of glomerular capillary is low
B. Plasma colloid osmotic pressure gradually increases along the glomerular capillary
C. Plasma crystal osmotic pressure gradually decreases along the glomerular capillary
D. Plasma crystal osmotic pressure gradually increases along the glomerular capillary
E. Hydrostatic pressure in Bowman’s capsular gradually increases
20. Key dynamic of the solute and water reabsorption at the proximal tubule is: A
  A. Na+-K+ pump
B. Carbonic anhydrase
C. Concentration gradient
D. Potential gradient
E. Osmotic pressure gradient
20. Site that is high permeable to water and impermeable to Na+ and Cl- is C
  A. Ascending limb thick segment of Henle’ s loop
B. Ascending limb thin segment of Henle’ s loop
C. Descending limb thin segment of of Henle’s loop
D. Descending limb thick segment of of Henle’s loop
E. Distal convoluted tubule
21. Site that can actively reabsorb Na+, Cl- and K+ but is impermeable to water is:  A
A. Ascending limb thick segment of Henle’ s loop
B. Ascending limb thin segment of Henle’ s loop
C. Descending limb thin segment of of Henle’s loop
D. Descending limb thick segment of of Henle’s loop
E. Collecting duct
22. Site that is high permeable to urea is:  D
  A. Distal convoluted tubule
B. Cortical segment of collecting duct
C. Outer medullary segment of collecting duct
D. Inner medullary segment of collecting duct
E. Descending limb thin segment of of Henle’s loop
23. Cause of polyuria (quantity of urine increases) in patient with diabetes mellitus mainly is: C
  A. Glumerular filtration rate increases
B. Colloid osmotic pressure of plasma decreases
C. Osmotic pressure of tubular fluid increases
D. Crystal osmotic pressure of plasma increases
E. Secretion of ADH decreases
24. Location in which Na+ is passively reabsorbed is: B
A. The first half of proximal tubule
B. The second half of proximal tubule
C. Ascending limb thick segment of Henle’ s loop
D. Distal convoluted tubule
E. Collecting duct
25. Mechanism forming osmolarity gradient in the outer medulla mainly is A
  A. NaCl reabsorption in the ascending limb thick segment of Henle’ s loop
B. Urea circulation
C. NaCl reabsorption in the descending limb thin segment of of Henle’s loop
D. NaCl reabsorption in the descending limb thick segment of of Henle’s loop
E. Urea reasorption in the ascending limb thick segment of Henle’ s loop
26. Which of the following descriptions about H+ secretion is wrong ? D
  A. Proximal tubule secretes H+ by H+-Na+ exchange (antiport of H+ and K+)
B. Intercalated cell in distal tubule and collecting duct secretes H+ via proton pump( H+- ATPase) 
C. Intercalated cell distal tubule and collecting duct secretes H+ via H+-K+ ATPase
D. When pH value of internal environment lowers, the activity of carbonic anhydrase decreases
E. H+ secretion promotes the reabsorption of HCO3- and the secretion of NH3
27. Which of the following descriptions about NH3 secretion is wrong  D
  A. NH3 derives mainly from the deamination of glutamine
B. NH3 is a lipid-soluble weak base
C. NH3 diffuses readily from epithelial cell into tubular fluid
D. NH3 secretion inhibits H+ secretion
E. NH3 secretion promote the reabsorption of HCO3-
28. Which of the following description about K+ secretion is wrong D
A. K+ in end-urine derives mainly from the secretion of distal convoluted tubule and collecting duct.
B. Principal cells secrete K+
C. Aldosterone promotes K+ secretion
D. Antidiuretic hormone inhibit K+ secretion
E. K+ secretion is a passive transport
29. Location in which the glomerulotubular balance occurs is:  A
  A. Proximal tubule.
B. Descending limb of Henle’s loop.
C. Ascending limb of Henle’s loop.
D. Henle’s loop.
E. Distal convoluted tubule and collecting duct.
30. Which of the following factors does not concerned with the osmotic diuresis E
  A. Increased solute concentration of tubular fluid.
B. Hyperosmotic glucose solution is given intravenously.
C. Mannitol is given intravenously.
D. Diabetes mellitus
E. Diabetes insipidus
31. Location in which water is regulatory reabsorbed is in:  E
  A. Proximal tubule
B. Descending limb of Henle’s loop
C. Ascending limb of Henle’s loop
D. Henle’s loop
E. Distal convoluted tubule and collecting duct
32. Normally, hormone regulating the reabsorption of water is:  D
  Normally, hormone maintaining the water balance of body is  D
A. Renin
B. Adrenaline (epinephrine)
C. Angiotensin Ⅱ
D. ADH ( antidiuretic hormone)
E. Aldosterone
33. Normally, the Na+ reabsorption and K+ secretion are regulated by: E
  Normally, hormone maintaining the Na+ and K+ balance is: E
A. Renin
B. Adrenaline (epinephrine)
C. Angiotensin Ⅱ
D. ADH ( antidiuretic hormone)
E. Aldosterone
34. After drinking a large amount of water, the reason for increased quantity of urine is that: C
  A. Glomerular filtration rate increases
B.. Aldosterone secretion decreases
C. ADH secretion decreases
D. Arterial blood pressure rises
E. Plasma colloid osmotic pressure decreases
35. After a large amount of sweating, the cause resulting in the decreased quantity of urine is that: C
A. Plasma crystal osmotic pressure declines
B. Plasma colloid osmotic pressure elevates
C. ADH secretion increases
D. Aldosterone secretion increases
E. Renal blood flow decreases
36. Main reason for that excitation of sympathetic nerve results in decreasing of urine quantity is: A
A. Renal blood flow decreases
B. Plasma colloid osmotic pressure elevate
C. ADH secretion increases
D. Aldosterone secretion decreases
E. Glomerular filtration rate increases
37. Location in which urine concentration occurs is:  D
A. Ascending limb thick segment of Henle’s loop
B. Distal convoluted tubule
C. Cortical segment of collecting duct
D. Medullary segment of collecting duct
E. Descending limb thin segment of Henle’s loop
38. Which of followings is not related to (concerned with) the urine dilution E
A. Ascending limb thick segment of Henle’ s loop
B. Distal convoluted tubule
C. Cortical segment of collecting duct
D. Inner medullary segment of collecting duct
E. Proximal tubule
39. Which of followings does not initiate ADH secretion ? B
  A. Dehydration (deficit of water)
B. Decreased plasma colloid osmotic pressure
C. Increased plasma crystal osmotic pressure
D. Severe pain
E. A large mount of hemorrhage 
40. Which of the followings does not initiate renin secretion ? E
  A. Reduction of afferent arteriolar transmural pressure
B. Reduction of NaCl flowing through the macula densa
C. Increased sympathetic nerve activity
D. Elevated plasma adrenaline and noradrenaline
E. Decreased intrarenal prostaglandin E2 formation.
41. Which of the followings does not cause aldosterone secretion ? D
  A. Decreased arterial blood pressure
B. Angiotensin Ⅱ
C. Angiotensin Ⅲ
D. Increased plasma Na+ concentration
E. Increased plasma K+ concentration
42 Normally, which of the following substances has the minimal renal clearance? A
  A. Glucose
B. Urea
C. Endogenous creatinine
D. Inulin
E. NaCl
Ⅱ. Define following word
1. Glomerular filtration When blood flows through the glomerular capillary, under driving of glomerular effective filtration pressure, the water and low-molecular weight solute in plasma move through filtration membrane into the Bowman’s capsule, this process is termed as glomerular filtration. The filtrate formed in glomerulus is ultrafiltrate, is also called the primary urine.
2. Glomerular filtration rate (GFR)  The quantity of ultrafiltrate formed by both kidneys per unit time(each minute) is called GFR. In normal adult human, GFR average 125ml /min, 180L/day.
3. Filtration fraction(FF) The ratio between glomerular filtration rate and renal plasma flow per a minute is termed FF.  FF = GFR/RPF(renal plasma flow). In normal human , FF is 19~20%
4. Glomerular effective filtration pressure (GEFP)
GEFP is the net dynamics of glomerular filtration.
    GEFP =  glomerular capillary hydrostatic pressure (GCHP) – glomerular capillary plasma colloid osmotic pressure ( GCOP) – Bowman’s capsular hydrostatic pressure
5. Autoregulation of renal blood flow (RBF)  When the arterial blood pressure is varied between 80 and 180 mmHg, the RBF, GCHP and GFR keep constant, which is called the autoregulation of RBF and GFR. 
6. Electrostatic barrier of glomerular filtration membrane.  The three layers of glomerular filtration membrane contain negatively charged glycoprotein, these negatively charged elements impede the passage of same charged plasma protein, which is called electrostatic barrier.
7. Filtration coefficient(Kf)  Kf means the quantity of filtrate per unit time and per unit GEFP
8. Filtration equilibrium  When the force opposing filtration become equal to the force driving filtration, the GEFP is zero, filtration ceases, which is called the filtration equilibrium.
9. Osmotic diuresis  When a large amount of solute are present in tubuar fluid, the increased osmotic pressure caused by the solute retards reabsorption of water and sodium, as a result, the quantity of urine increases, which is called osmotic diuresis.
10. Glomerulotubular balance  The proximal tubular always reabsorbs the 65~70% of filtrated water and Na+ in despite of the GFR increasing and decreasing, this phenomenon is called glomerulotubular balance.
11. Water diuresis  Drinking a large amount of water causes the quantity of urine to increase, which is called water diuresis.
12. renal clearance(Cx) The renal clearance of a substance is the volume of plasma that is completely cleared of this substance by kidneys per unit time.

Ⅲ. Answer following question
1. Describe the characteristics of renal blood circulation.
Answer:
(1). Double capillary bed: The renal blood circulation has two capillary bed in series, glomerular capillary and peritubular capillary. The glomerular capillary has an high hydrostatic pressure (blood pressure), which is favorable to fluid filtration. The peritubular capillary have lower hydrostatic pressure and higher plasma colloid osmotic pressure, which is favorable to fluid reabsorption.
(2). Renal blood flow (RBF) is large. RBF of two kidney in a resting, young man,is about 1200ml /min, is about 20~25% of cardiac output.
(3). RBF is not uniformly distributed. Renal cortex receives 94% of RBF. This blood flow is necessary to sustain a high glomerular filtration rate (GRF). Outer marrow receives 5% of RBF . Inner marrow receives 1% of RBF. The relative low blood flow in the marrow preserves the osmotic gradient in this region.
(4). Characteristics of regulation of RBF
1) Autoregulation: When the arterial blood pressure is varied between 80 and 180 mmHg, the RBF, GCHP and GFR keep constant, which is called the autoregulation of RBF and GFR. 
2) Nervous regulation and humoral regulation:
Sympathetic nerve: The renal sympathetic nerves innervate the renal vascular smooth muscle. In response to the state of emergency (critical condition), the increased activity of sympathetic nerve causes the renal blood vessel to constrict, RBF decreases.
Adrenaline (epinephrine ) and noradrenaline (norepinephrene ) causes the renal blood vessel to constrict, RBF decreases.
Prostaglandin E2 (PGE) , prostacyclin(PGI2), NO, bradykinin, atrial natriuretic peptide (ANP) cause renal vessel to relax, RBF increasing.
2. What is glomerular filtration membrane? What are characteristics of glomerular filtration membrane (GFM)?
Glomerular filtration membrane means the membranous structure between the glomerular capillary blood and Bowman’s capsula cavity.     
  GFM consists of three layers- capillary endothelium, basement membrane, and epithelial  cell(podocytes). They are both structural base and barrier of glomerular  filtration,
  (1). The capillary endothelial cell layer has many round, window-like holes , which is called fenestrea , its diameter is about 50~100 nm. Because the diameter of fenestrea is relatively large, the endothelial layer is not a major barrier for filtration.
(2). The basement membrane consist of meshwork of fine fibrils ( collagen and proteoglycan fibrils) embedded in a gel-like matrix. The basement membrane contains mesh holes about 2~8 nm in diameter. The mesh holes prevents effectively filtration of plasma protein, and is major barrier for plasma protein.
(3). The epithelial cell layer consist of “foot cell ” or podocytes. The epithelial cell layer contains slit pores about 4~12 nm in diameter. 
Accordingly, GFM is quite porous, has high permeability to water and low-molecules solute, but restricts passage of the plasma protein.
The three layers of GFM contain negatively charged glycoprotein, these negatively charged elements impede the passage of same charged plasma protein, which is called electrostatic barrier.
3. Describe factors determining glomerular filtration rate.
Answer:
(1). GEFP (glomerular effective filtration pressure):
1) GCHP (glomerular capillary hydrostatic pressure): The GCHP is the only driving force for filtration. GCHP is determined by the arterial blood pressure and the contractile state of afferent and efferent arteriole. .
  When the arterial blood pressure is varied between 80 and 180 mmHg, the RBF, GCHP and GFR keep constant, which is called the autoregulation of RBF and GFR. 
If arterial pressure is lowered below 80mmHg, the GCHP will markedly fall, and thus GFR will reduce. 
If arterial pressure is declined below 50mmHg, the GEFP falls to zero, GFR is zero, the patient or animal will be anuria.
  When constriction of the afferent arteriole strengthen, the precapillary resistance increase; GCHP declines, GEFP decreases; GFR decreases. When the constriction of efferent arteriole strengthen, postcapillary resistance increases; GCHP elevates; GFR increases.
2) GCOP (glomerular capillary plasma colloid osmotic pressure): The GCOP is force opposing filtration; normally, it is stable.
With intravenous infusion of a large volume of normal saline, a decreased GCOP increases the GEFP and GFR.
3) Boeman’s capsule hydrostatic pressure: Boeman’s capsule hydrostatic pressure is the force opposing filtration. Normally, Bowman’s capsule hydrostatic pressure is stable. In certain pathological state, such as obstruction of urinary tract, Bowman’s capsule pressure increases, GFR reduces. 
  (2). Renal plasma flow (RPF)
  GFR is in direct proportion to RPF.
Mechanism : As mention above, the GCOP will increase due to prior filtration. When RPF increases, the increase of GCOP along the length of glomerular capillary will become slow, the location at which the filtration equilibrium is achieved nears the efferent arteriolar end. The effective filtration membrane area increases, GFR increases. Conversely, if the RPF decreases, GFR decreases.
  (3). Filtration coefficient ( KF) 
  GFR is in direct proportion to Kf.  GFR = Kf × GEFP
Kf means the quantity of filtrate per unit time and per unit GEFP .
Kf is in direct proportion to both the fluid permeability and surface area of filtration membrane.
If there is a decrease in either permeability or surface area of filtration membrane, GFR will decrease. In chronic renal  disease, the glomeruli are destroyed, leading to a reduction in filtration membrane area and a diminished GFR. If some diseases damage the mechanical and electrostatic barrier, the permeability of filtration membrane increases, leading to hematuria and  proteinuria.
4. When a person have a large amount of haemorrhage and thus his arterial blood pressure decreases markedly, how change his quatity of urine? Why?
  Answer:
    His quantity of urine decreses.
    Mechanism:
(1) GFR decreses: The declined excessively arterial blood pressure result in the glomerular capillary hydrostatic pressure to decline, the glomerular effective filtration pressure reducing, glomerular filtration rate (GFR) decreasing.
(2) Renal plasma flow decreases: Both declined arterial blood pressure and the activity of sympathetic nerve causes the renal plasma flow to decrease; the increase of the plasma colloid osmotic pressure along the length of glomerular capillary will become quick, the location at which the filtration equilibrium is achieved nears the afferent arteriolar end. Consequently, effective filtration membrane area decreases, GFR decreases.
(3) Release of ADH increases:The stimulation of the decreased excessively blood volume to the volume receptors located on left atrium and thoracic large vines decreases; the afferent signals of the vagus nerve decreases; the release of ADH increases reflexly. The increased ADH plasma concentration elevates the permeability of distal convoluted tubule and collecting duct to water, thereby increasing water reabsorption, the quantity of urine decreasing and urine concentrates.
(4) Release of aldosterone increases: The decreased blood pressure at the afferent arteriole and the excessive activity of sympathetic nerve causes the juxtaglomerular cell to release rennin. The rennin-angiotensin-aldosterone system is started. Release of aldosterone increases. The aldosterone promotes the distal convoluted tubule and collecting duct to reabsorb Na+ and water. 
5. Describe the process of reabsorption of N+ and Cl- at proximal tubule. 
Answer:
The N+ - K+ pump located on basolateral membrane of epithelial cell of the proximal tubule actively transports Na+ from cell to interstitial fluid and K+ from interstitial fluid into cell. The operation of Na+-K+ pump carries out the reabsorption of Na+, maintains the low Na+ and the high K+ of intracellurar fluid and creates a negative potential within cell. This electrochemical gradient will provides energy for symport , antiport .
In the apical membrane of epithelial cell of the first half of proximal tubule, Na+ is reabsorbed mainly by symport along with glucose and by antiport with H+ from tubular fluid into epithelial cell;HCO3- is preferentially reabsorbed, Cl- is not reabsorbed. Therefore, the Cl- concentration increases gradually. In second half of proximal tubule, the Cl- diffuse from tubular lumen into intercellular fluid via the tight junction down concentration difference, creating a transtubular potential difference that is positive in tubular lumen and is negative in interstitial fluid, this potential difference causes Na+ to diffuse into interstitial fluid. This is passive transport..
6. Describe the process of reabsorption of N+ and Cl-  at thick segment of Henle’s loop ascending limb
    Answer:
The N+ - K+ pump located on basolateral membrane of epithelial cell of the thick segment of Henle’s loop ascending limb actively transports Na+ from cell to interstitial fluid and K+ from interstitial fluid into cell. The operation of Na+-K+ pump carries out the reabsorption of Na+, maintains the low Na+ and the high K+ of intracellurar fluid and creates a negative potential  within cell. This electrochemical gradient will provides energy for symport , antiport .
There are a symporter of Na+, K+, Cl- on apical membrane of the cell of the thick segment ascending limb; Cl-. Na+, K+, Cl- combine with the sympoter in the proportion of 1-Na+:2-Cl-:1K+, then the Na+ and Cl- moves down its electrochemical gradient, while K+ against their electrochemical gradient from tubular fluid into the cell.
The increased intracellular Na+ is actively transported by Na+-K+ pump on basolateral membrane into interstitial fluid, the Cl- diffuses passively into interstitial fluid, the K+ partly into interstitial fluid and partly diffuses back tubular fluid and thus cause the tubular fluid to be positive potential. The positive potential in tubular fluid promotes Na+, K+ and Ca2+ to be passively reabsorbed via the paracellular transport (50%). 
7. Describe the process of reabsorption of N+ and Cl-  at the distal convoluted tubule and collecting duct.
  Answer:
The Na+ - K+ pump located on basolateral membrane of the principal cell of the distal convoluted tubule and collecting duct actively transports Na+ from cell to interstitial fluid and K+ from interstitial fluid into cell. The operation of Na+-K+ pump carries out the reabsorption of Na+, maintains the low Na+ and the high K+ of intracellurar fluid and creates a negative potential within cell. This electrochemical gradient will provides energy for symport of Na+ - Cl-.
There are a symporter of Na+ - Cl- on the luminal membrane of of the initial segment of distal convoluted tubule. The Na+ and Cl- are transported across the luminal membrane from tubular fluid into cell by the symport.
  There are a Na+ channel on the luminal membrane of principal cell in the latter second half segment of distal convoluted tubule and collecting duct. The Na+ in tubular fluid diffuse into cell through the Na+ channel, and thus creates a negative potential in the tubular fluid. The negative potential drives the Cl- reabsorption via paracelluar transport and K+ secretion.
8. Describe the identical points and different points of reabsorption of N+ and Cl- at proximal tubule, thick segment of Henle’s loop ascending limb, distal convoluted tubule and collecting duct.
  Answer:
  The identical points: The N+ - K+ pump located on basolateral membrane of epithelial cell of the proximal tubule, distal tubule, and collecting duct primarily actively transports Na+ from cell to interstitial fluid and K+ from interstitial fluid into cell. The operation of Na+-K+ pump carries out the reabsorption of Na+, maintains the low Na+ and the high K+ of intracellurar fluid and creates a negative potential of about -70 mv within cell. This electrochemical gradient will provides energy for symport , antiport . Reabsorption of Cl- is passive transport.
  The different points:
① In the epithelial cell apical membrane of the first half of proximal tubule, Na+ is reabsorbed mainly by symport along with glucose and by antiport with H+ from tubular fluid into epithelial cell;HCO3- is preferentially reabsorbed, Cl- is not reabsorbed. Therefore, the Cl- concentration increases gradually. In second half of proximal tubule, the Cl- diffuse from tubular lumen into intercellular fluid via the tight junction down concentration difference, creating a transtubular potential difference that is positive in tubular lumen and is negative in interstitial fluid, this potential difference causes Na+ to diffuse into interstitial fluid. This is passive transport..
②The apical membrane of the cell of the thick segment ascending limb has a symporter of Na+, K+, Cl-. Na+, K+, Cl- combine with the sympoter in the proportion of 1-Na+:2-Cl-:1K+, then the Na+ and Cl- moves down its electrochemical gradient, while K+ against their electrochemical gradient from tubular fluid into the cell. The increased intracellular Na+ is actively transported by Na+-K+ pump on basolateral membrane into interstitial fluid, the Cl- diffuses passively into interstitial fluid, the K+ partly into interstitial fluid and partly diffuses back tubular fluid and thus cause the tubular fluid to be positive potential. The positive potential in tubular fluid promotes Na+, K+ and Ca2+ to be passively reabsorbed via the paracellular transport (50%).
③There are a symport Na+ - Cl- on the luminal membrane of of the initial segment of distal convoluted tubule. The Na+ and Cl- are transported across the luminal membrane from tubular fluid into cell by the symport.
  There are a Na+ channel on the luminal membrane of principal cell in the latter second half segment of distal convoluted tubule and collecting duct.  The Na+ in tubular fluid diffuse into cell through the Na+ channel, and thus creates a negative potential in the tubular fluid. The negative potential drives the Cl- reabsorption via paracelluar transport and K+ secretion.
9. Describe Na+-glucose symport and Na+-H+ antiport in the proximal tubule.
Answer:
The Na+ - K+ pump located on basolateral membrane of epithelial cell of the proximal tubule, distal tubule, and collecting duct transports Na+ from cell to interstitial fluid and K+ from interstitial fluid into cell. The operation of Na+-K+ pump carries out the reabsorption of Na+, maintains the low Na+ and the high K+ of intracellurar fluid and creates a negative potential  within cell. This electrochemical gradient will provides energy for symport and antiport .
In the epithelial cell luminal membrane (apical membrane) of proximal tubule, Na+ is reabsorbed by symport along with glucose and by antiport with H+, from tubular fluid into epithelial cell.  Na+-glucose symport: The Na+ and glucose of tubular fluid bind with the same co-transporter of luminal membrane, then, the Na+ moves into the cell down its electrochemical gradient while the glucose is pulled into cell against its concenteation difference. Then, this increased intracellular glucose diffuse via facilitated diffusion via carrier into interstitial fluid and blood.
Na+-H+ antiport : The intracellular H+ and Na+ in tubular lumen bind with the same antiporter of luminal membrane, then, the Na+ move into cell down its electrochemical gradient while the H+ is pushed out of cell against its electrochemical gradient into tubular lumen, which is called H+-Na+ exchange.
10.describe the process of reabsorption of water in the renal tubule and collecting duct.
Answer:
The manner of water reabsorption is osmosis, is passive transport. The direct driving force for water reabsorption is the osmolarity gradient between interstitial fluid and tubular fluid. The osmolarity (or osmotic pressuere) of the interstitial fluid is higher than that of the tubular fluid, then water penetrates down the osmolarity gradient from the tubular fluid to the interstitial fluid.  This osmolarity gradient result from the reabsorption of Na+, Cl- ,and other solutes. Therefore, reabsorption of water is directly related to the reabsorption of solute. The proximal tubule is high permeable to water. The reabsorption of water in proximal tubule is iso-osmotic reabsorption. The descending limb of Henle’s loop is high permeable to water, water diffusing from tubular fluid to medulla interstitial fluid and thus causing the osmolarity of tubular fluid to be increased. The ascending limb of Henle’s loop is impermeable to water, water can not being reabsorbed at this portion. The reabsorption of water in proximal tubule and Henle’s loop is relatively fixed and is not affected by regulatory factors.
The reabsorption of water at distal convoluted tubule and collecting duct is regulated by ADH. In the presence of high concentration ADH, the permeability of these tubule to water increases markedly, water being reabsorbed. The increased water and solutes in the intercellular fluid elevates the hydrostatic pressure in this region and thereby drives fluid into peritubular capillary.
11. Describe the process of secretion of K+ in distal convoluted and collecting duct.
Answer:
The vast majority of filtrated K+ is reabsorbed in proximal tubule and Henle’s loop. The K+ in end-urine derives mainly from secretion of principal cell in distal convoluted tubule and collecting duct.
The positions of K+ secretion are the distal convoluted tubule and collecting duct.
The manner of K+ secretion is a passive transport.
The driving force of K+ secretion is the negative potential in tubular fluid created by the active reabsorption of Na+.
The Na+-K+ pump located on basolateral membrane of principal cell of distal convoluted tubule and collecting duct actively transports Na+ from cell into interstitial fluid and causes the high K+ and low Na+ of the intracellular fluid. Na+ of tubular fluid diffuses down electrochemical gradient into cell, and thus creates a negative potential in the tubular fluid. This negative potential and increased intracellular K+ drives K+ from cell into tubular fluid. Therefore , K+ secretion is positively related to the activity of Na+-K+ pump and the permeability of luminal membrane to Na+.
12. Describe the process of H+ secretion and HCO3- reabsorption  in renal tubule, collecting duct and deacribe its sinificance.
In proximal tubule and thick segment of Henle’s loop ascending limb, H+ is secreted by Na+-H+ exchange.
Na+-H+ antiport : The intracellular H+ and Na+ in tubular lumen bind with the same antiporter of luminal membrane, then, the Na+ move into cell down its electrochemical gradient while the H+ is pushed out of cell against its electrochemical gradient into tubular lumen, which is also called H+-Na+ exchange. The H+ secretion couples with the reabsorption of HCO3-.
The HCO3-(bicarbonate) filtrated can not permeate the luminal membrane, therefore HCO3- can not be directly reabsorbed.   
In tubular fluid  Carbonic
Anhydrase of
apical membrane
,                HCO3- + H+ ←  ——— →  H2CO3 → H2O +CO2
The CO2 diffuses down its concentration difference into tubular cell .
            In tubular cell  Carbonic
anhydrase
              CO2 + H2O ←———→ H2CO3 → H+ + HCO3-
The majority of HCO3- are transported along with Na+ into interstitial fluid and then blood, while the H+ is actively secreted into tubular fluid by Na+-H+ exchange.
Therefore, the manner of HCO3- reabsorption is CO2 .
In the distal tubule and collecting duct, the intercalated cell actively secretes H+, its process is following:
There are much CO2  in the intercalated cells. These CO2 are from both the metabolism of cell and blood.
        In intercalated cell  Carbonic
anhydrase
              CO2 + H2O ←———→ H2CO3 → H+ + HCO3-     
The HCO3- is reabsorbed into interstitial fluid, while the H+ is actively transported by proton pump( H+- ATPase)and H+-K+ ATPase into tubular fluid.
Significance: ①Whenever a H+ is secreted , there must be a HCO3- is generated or reabsorbed. Kidneys maintains the acid-base balance of internal environment by reabsorbing HCO3- and secreting H+. When pH value of internal environment lowers, the activity of carbonic anhydrase increases, secretion of H+ and reabsorption of HCO3- increasing. .② It promotes NH3 secretion. H+ in tubular fluid bind with NH3 to form NH4+. NH4+concentration of tubular fluid decreasing, NH3 secretion increasing.
13. Giving intravenously hyperosmotic glucose solution to a animal (or person), how changes its (or his ) quantity of urine ? why ?
Answer:
Giving intravenously hyperosmotic glucose solution to a animal (or person) leads to the osmotic diuresis. The quantity of urine increases.
Mechanesm: When the hyperosmotic glucose solution is injected intravenously, the blood glucose concentration rises excessively and is higher than the renal glucose threshold. The glucose can not be reabsorbed completely. The excessive glucose remains in the tubular fluid, the osmotic pressure of tubular fluid increases, which retards water reabsorption, the quantity of urine  increasing. For the same reason, the quantity of urine of the most of parients with diabetes increases(hyperuria).
14. When a person is subjected a large amount of sweating and deprivation of drinking water, how change his quantity of urine? Way?
Answer:
His quantity of urine decreases.
Mechnism: The reasons for this are: (1) His plasma crystal osmotic pressure increases. The increased  plasma crystal osmotic pressure stimulates the osmoreceptor in hypothalamus and thus causes posterior pituitary to release ADH. The increased ADH plasma concentration elevates the permeability of distal convoluted tubule and collecting duct to water, thereby increasing water reabsorption, the quantity of urine decreasing and urine concentrates.
(2) His blood volume decrease. The stimulation of the decreased blood volume to the volume receptors located on left atrium and thoracic large vines (such as vena cava, pulmunary vines) decreases; the afferent signals of the vagus nerve decreases; the release of ADH increases reflexly. Finally, the quantity of urine decreases for the same reason.
Conversely, when a person drinks a large amount of water, the opposite sequence of events occurs.
15. When a person drinks a large amount of water, how change his quantity of urine? Way?
  Answer:
  His quantity of urine increases
  Mechanism: (1) After drinking a large amount of water, his plasma is diluted. The stimulation of decreased plasma crystal osmotic pressure to the osmoreceptor in hypothalamus weaken and thus causes the release of ADH to reduce. The decreased ADH plasma concentration reduces the permeability of distal convoluted tubule and collecting duct to water. The reabsorption of water decreases, the quantity of urine increasing.
  (2) After drinking a large amount of water, his blood volume increases. The increased blood volume stimulates the volume receptors located on left atrium and thoracic large vines, the afferent signals of the vagus nerve increasing, the release of ADH reducing reflexly. For the same reason, the quantity of urine increases.   
16. Describe the function and its functional mechanisms of aldosterone on renal tubule and collecting duct
The aldosterone promotes the Na+ reabsorption and K+ secretion in distal convoluted tubule and collecting duct.
Mechanism: Aldosterones enter the principal cells; bind with the receptor in cytoplasm to form a hormone-cytoplasm receptor complex. The complex are transported into the nucleus, then bind with the receptor in the nucleus to form hormone-nucleus receptor complex and thus activates the transcription(转录)process of specific genes to form specific mRNA. The mRNA diffuses into cytoplasm, where it promotes the translation(转化)process to form aldosterone-induced proteins. The latter ① increase the number of the Na+ channels and thus increase the Na+ permeability of the luminal membrane , ② enhance the activity of the Na+ - K+ ATPase pump on the basolateral membrane, ③ increase the synthesis of ATP.
17. Describe the mechanism by which the renal medullary osmolarity gradient formed.
(1). Mechanism forming osmolarity gradient in outer medulla
In outer medulla, the increased osmolarity of interstitial fluid result mainly from the active reabsorption of Na+ and Cl-, while water can not be reabsorbed in the ascending limb thick segment of Henle’s loop. The ascending limb thick segment of Henle’ s loop is impermeable to water, actively reabsorbs Na+ and Cl-, consequently, the NaCl concentration in interstitium is increased, rising osmolarity of interstitial fluid in outer medulla .
(2) . Mechanism forming osmolarity gradient in inner medulla
In inner medulla, the increased concentration of NaCl and urea contribute equally to the increased osmolarity of interstitial fluid .
The process forming the osmolarity gradient of inner medulla is countercurrent multiplication,  is following:
1)The descending limb thin segment of of Henle’s loop is impermeable to urea and NaCl, is highly permeable to water. As tubular fluid flow through this region, water is net reabsorbed, the concentration of NaCl in tubular fluid progressively increases.
2)When this tubular fluid rounds the bend of Henle’s loop and flow into ascending limb thin segment, because the ascending limb thin segment is highly permeable to NaCl and is impermeable to water, NaCl diffuses down concentration difference from tubular fluid into inner medullary peritubular fluid, thereby increasing its osmolarity.
3) Urea circulation increases the urea concentration of inner medulla
The distal convoluted tubule and cortical segment and outer medullary segment of collecting duct is impermeable to urea. In the presence of high concentration ADH, water is reabsorbed from these segments, increasing urea concentration in tubular fluid.
Because the inner medullary collecting duct is highly permeable to urea, then urea diffuses down its concentration difference from tubule into peritubular interstitium and thus increase the urea concentration and osmolarity of peritubular fluid in the inner medulla.
The urea in inner medulla diffuses into ascending limb thin segment, then flow through ascending limb thick segment, distal convoluted tubule, outer medullary segment of collecting duct , and finally back to the inner medullary collecting duct where urea diffuses into interstitial fluid again, this process is called urea circulation .
18. Describe the process of urine dilution and concentration.
(1) Urine dilution
In the ascending limb thick segment of Henle’s loop, Na+, K+, Cl- are avidly reabsorbed, however, this portion of tubule is impermeable to water, therefore the tubular fluid become dilute , its osmolarity is about 100mmol/L. As these dilute fluid flows through the distal tubule and collecting duct, the Na+, K+,Cl- is continuously reabsorbed. In the absence of ADH, these tubule are also impermeable to water. Therefore, the tubular fluid become even more dilute, its osmolarity lowers as low as 50 mOsm/L.
(2) Urine concentration.
The urine concentration occurs in the medullary segment of collecting duct. In presence of high concentration ADH, the distal convoluted tubule and collecting duct is high permeable to water. When tubular fluid flow through the medallary segment of collecting duct, because there is a osmolarity gradient in the interstitial fluid of renal medalla, water moves down osmotic pressure difference from tubular fluid into renal medullary interstitial fluid, up to osmotic equilibrium is achieved, consequently, the quantity of urine decreases and urine osmolarity increases.
离线孙笑天
发帖
97
啄木币
1729
鲜花
82
只看该作者 2楼 发表于: 2007-12-15
                          英文题  第八章 (2006. 3. 23)
Ⅰ. 选择题
1. The main difference between primary urine and blood plasma is:  D
  A. Glucose concentration.
B. Crystal osmotic pressure.
C. NaCl concentration.
D. Plasma protein concentration.
E. pH value.
2. Site of renin secretion is at  A
  A. Juxtaglumerular cells.
B. Extraglomerular mesangial cells.
C. Juxtaglomerular apparatus.
D. Macula densa.
E. Glomerular capillary
3. Macula densa is a  A
A. Chemical receptor.
B. Baroreceptor.
C. Stretch receptor.
D. Volume receptor.
E. Osmoreceptor.
4. Characteristic of renal blood circulation is:  A
  A. Double capillary beds.
B. Low capillary hydrostatic pressure.
C. High peritubular capillary hydrostatic pressure.
D. Smaller blood flow of renal cortex.
E. Greater blood flow of renal marrow (medulla).
5. Which of the following descriptions about urine of a normal adult is true ? D
  A. Daily urine quantity is 2 to 3 L/ day.
B. Its osmotic pressure is equal to that of plasma.
C. It contains a large amount of NaCl.
D. Urine does not contain glucose.
E. Urine contains protein.
6. Which of the followings can increase the glomerular filtration rate ? C
  A. Arterial blood pressure rises from 80 to 180 mmHg.
B. Arterial blood pressure declines from 180 to 80 mmHg.
C. A large amount of normal saline is given intravenously.
D. Excitation of sympathetic nerve.
E. Constriction of afferent arteriole.
7. The driving force for glumerular filtration is:  A
  A. Glomerular capillary hydrostatic pressure.
B. Arterial blood pressure.
C. Plasma colloid osmotic pressure.
D. Bowman’s capsular hydrostatic pressure.
E. Blood pressure of efferent arteriole.
8. The main barrier of glomerular filtration membrane is located on B
  A. Capillary endothelial cell layer.
B. Basement membrane.
C. Visceral epithelial cell (“foot cell ” or podocyte) layer.
D. Parietal epithelial cell layer.
E. Negatively charged elements of capillary endothelial cell layer.
9. If some diseases damage the negatively charged elements of glomerular membrane, which of the following will occur ? B
  A. Glomerular filtration rate increases.
B. Proteinuria (protein in urine).
C. Hematuria ( hematuresis or blood in the urine).
D. Renal blood flow increases.
E. Plasma protein concentration increases.
10. Glucose concentration of the ultrafiltrate (primary urine) is: A
  A. Similar to blood plasma.
B. Higher than that of plasma.
C. Lower than that of plasma.
D. Similar to urine.
E. Similar to tubular fluid in distal tubule.
11. Normally, which of following substances can not move through (pass across or penetrate) glomerular filtration membrane? A
  A. Plasma protein 
B. Glocose
C. Creatinin
D. urea
E. Inulin
12. When noradrenaline (norepinephrine ) is given intravenously, the main cause of urine quantity decreasing is: B
  A. Boeman’s capsule hydrostatic pressure increases.
B. Renal arteriole constricts. 
C. Release of ADH increases.
D. Release of renin decreases.
E. Release of aldosterone decrease.   
13. Which of the following descriptions about glomerular filtration membrane is wrong?  E
  A. It consists of capillary endothelium, basement membrane, and epithelial cell.
B. It is quite porous.
C. It is structural base of glomerular filtration.
D. The basement membrane is major barrier for plasma protein.
E. The negatively charged protein move easily through the filtration membrane.
14. If reabsorption of renal tubule and collecting duct to water decreases 1%, the quantity of urine will increase  D
  A. 1%
B. 2%
C. 50%
D. 100%
E. 300%
15.Which of the following descriptions about the reabsorption of Na+ and Cl- at the proximal
tubule is wrong?  E
A. The intracellular Na+ are transported by Na+ - K+ pump to interstitial fluid   
B. The process by which Na+ move from tubular fluid into cell is passive transport. 
C. In second half of proximal tubule, Na+ and Cl-diffuse from tubular lumen into intercellular fluid mainly via the tight junction.
D. The first half of proximal tubule does not reabsorb Cl- .
E. It is regulated by aldosterone
16. Proximal tubule of kidneys reabsorbs  C
  A. 85% of filtrated water.
B. 85% of filtrated Na+.
C. 65~70% of filtrated Na+ and water.
D. 65~70% of filtrated glucose.
E. 65~70% of filtrated HCO3-.
17. Location reabsorbing glucose is at: A
  A. Proximal tubule.
B. Descending limb of Henle’ loop.
C. Ascending limb of Henle’ loop.
D. Distal convoluted tubule.
E. Collecting duct.
18. Normally, proximal tubule reabsorbs:  E
  A. 20% of filtrated glucose.
B. 50% of filtrated glucose.
C. 65~70% of filtrated glucose.
D. 80% of filtrated glucose.
E. 100% of filtrated glucose.
19. The reason for that the filtration occurs only in afferent
arteriole end of the glomerular capillary is:  B
  A. Blood pressure at the efferent arteriole end of glomerular capillary is low
B. Plasma colloid osmotic pressure gradually increases along the glomerular capillary
C. Plasma crystal osmotic pressure gradually decreases along the glomerular capillary
D. Plasma crystal osmotic pressure gradually increases along the glomerular capillary
E. Hydrostatic pressure in Bowman’s capsular gradually increases
20. Key dynamic of the solute and water reabsorption at the proximal tubule is: A
  A. Na+-K+ pump
B. Carbonic anhydrase
C. Concentration gradient
D. Potential gradient
E. Osmotic pressure gradient
20. Site that is high permeable to water and impermeable to Na+ and Cl- is C
  A. Ascending limb thick segment of Henle’ s loop
B. Ascending limb thin segment of Henle’ s loop
C. Descending limb thin segment of of Henle’s loop
D. Descending limb thick segment of of Henle’s loop
E. Distal convoluted tubule
21. Site that can actively reabsorb Na+, Cl- and K+ but is impermeable to water is:  A
A. Ascending limb thick segment of Henle’ s loop
B. Ascending limb thin segment of Henle’ s loop
C. Descending limb thin segment of of Henle’s loop
D. Descending limb thick segment of of Henle’s loop
E. Collecting duct
22. Site that is high permeable to urea is:  D
  A. Distal convoluted tubule
B. Cortical segment of collecting duct
C. Outer medullary segment of collecting duct
D. Inner medullary segment of collecting duct
E. Descending limb thin segment of of Henle’s loop
23. Cause of polyuria (quantity of urine increases) in patient with diabetes mellitus mainly is: C
  A. Glumerular filtration rate increases
B. Colloid osmotic pressure of plasma decreases
C. Osmotic pressure of tubular fluid increases
D. Crystal osmotic pressure of plasma increases
E. Secretion of ADH decreases
24. Location in which Na+ is passively reabsorbed is: B
A. The first half of proximal tubule
B. The second half of proximal tubule
C. Ascending limb thick segment of Henle’ s loop
D. Distal convoluted tubule
E. Collecting duct
25. Mechanism forming osmolarity gradient in the outer medulla mainly is A
  A. NaCl reabsorption in the ascending limb thick segment of Henle’ s loop
B. Urea circulation
C. NaCl reabsorption in the descending limb thin segment of of Henle’s loop
D. NaCl reabsorption in the descending limb thick segment of of Henle’s loop
E. Urea reasorption in the ascending limb thick segment of Henle’ s loop
26. Which of the following descriptions about H+ secretion is wrong ? D
  A. Proximal tubule secretes H+ by H+-Na+ exchange (antiport of H+ and K+)
B. Intercalated cell in distal tubule and collecting duct secretes H+ via proton pump( H+- ATPase) 
C. Intercalated cell distal tubule and collecting duct secretes H+ via H+-K+ ATPase
D. When pH value of internal environment lowers, the activity of carbonic anhydrase decreases
E. H+ secretion promotes the reabsorption of HCO3- and the secretion of NH3
27. Which of the following descriptions about NH3 secretion is wrong  D
  A. NH3 derives mainly from the deamination of glutamine
B. NH3 is a lipid-soluble weak base
C. NH3 diffuses readily from epithelial cell into tubular fluid
D. NH3 secretion inhibits H+ secretion
E. NH3 secretion promote the reabsorption of HCO3-
28. Which of the following description about K+ secretion is wrong D
A. K+ in end-urine derives mainly from the secretion of distal convoluted tubule and collecting duct.
B. Principal cells secrete K+
C. Aldosterone promotes K+ secretion
D. Antidiuretic hormone inhibit K+ secretion
E. K+ secretion is a passive transport
29. Location in which the glomerulotubular balance occurs is:  A
  A. Proximal tubule.
B. Descending limb of Henle’s loop.
C. Ascending limb of Henle’s loop.
D. Henle’s loop.
E. Distal convoluted tubule and collecting duct.
30. Which of the following factors does not concerned with the osmotic diuresis E
  A. Increased solute concentration of tubular fluid.
B. Hyperosmotic glucose solution is given intravenously.
C. Mannitol is given intravenously.
D. Diabetes mellitus
E. Diabetes insipidus
31. Location in which water is regulatory reabsorbed is in:  E
  A. Proximal tubule
B. Descending limb of Henle’s loop
C. Ascending limb of Henle’s loop
D. Henle’s loop
E. Distal convoluted tubule and collecting duct
32. Normally, hormone regulating the reabsorption of water is:  D
  Normally, hormone maintaining the water balance of body is  D
A. Renin
B. Adrenaline (epinephrine)
C. Angiotensin Ⅱ
D. ADH ( antidiuretic hormone)
E. Aldosterone
33. Normally, the Na+ reabsorption and K+ secretion are regulated by: E
  Normally, hormone maintaining the Na+ and K+ balance is: E
A. Renin
B. Adrenaline (epinephrine)
C. Angiotensin Ⅱ
D. ADH ( antidiuretic hormone)
E. Aldosterone
34. After drinking a large amount of water, the reason for increased quantity of urine is that: C
  A. Glomerular filtration rate increases
B.. Aldosterone secretion decreases
C. ADH secretion decreases
D. Arterial blood pressure rises
E. Plasma colloid osmotic pressure decreases
35. After a large amount of sweating, the cause resulting in the decreased quantity of urine is that: C
A. Plasma crystal osmotic pressure declines
B. Plasma colloid osmotic pressure elevates
C. ADH secretion increases
D. Aldosterone secretion increases
E. Renal blood flow decreases
36. Main reason for that excitation of sympathetic nerve results in decreasing of urine quantity is: A
A. Renal blood flow decreases
B. Plasma colloid osmotic pressure elevate
C. ADH secretion increases
D. Aldosterone secretion decreases
E. Glomerular filtration rate increases
37. Location in which urine concentration occurs is:  D
A. Ascending limb thick segment of Henle’s loop
B. Distal convoluted tubule
C. Cortical segment of collecting duct
D. Medullary segment of collecting duct
E. Descending limb thin segment of Henle’s loop
38. Which of followings is not related to (concerned with) the urine dilution E
A. Ascending limb thick segment of Henle’ s loop
B. Distal convoluted tubule
C. Cortical segment of collecting duct
D. Inner medullary segment of collecting duct
E. Proximal tubule
39. Which of followings does not initiate ADH secretion ? B
  A. Dehydration (deficit of water)
B. Decreased plasma colloid osmotic pressure
C. Increased plasma crystal osmotic pressure
D. Severe pain
E. A large mount of hemorrhage 
40. Which of the followings does not initiate renin secretion ? E
  A. Reduction of afferent arteriolar transmural pressure
B. Reduction of NaCl flowing through the macula densa
C. Increased sympathetic nerve activity
D. Elevated plasma adrenaline and noradrenaline
E. Decreased intrarenal prostaglandin E2 formation.
41. Which of the followings does not cause aldosterone secretion ? D
  A. Decreased arterial blood pressure
B. Angiotensin Ⅱ
C. Angiotensin Ⅲ
D. Increased plasma Na+ concentration
E. Increased plasma K+ concentration
42 Normally, which of the following substances has the minimal renal clearance? A
  A. Glucose
B. Urea
C. Endogenous creatinine
D. Inulin
E. NaCl
Ⅱ. Define following word
1. Glomerular filtration When blood flows through the glomerular capillary, under driving of glomerular effective filtration pressure, the water and low-molecular weight solute in plasma move through filtration membrane into the Bowman’s capsule, this process is termed as glomerular filtration. The filtrate formed in glomerulus is ultrafiltrate, is also called the primary urine.
2. Glomerular filtration rate (GFR)  The quantity of ultrafiltrate formed by both kidneys per unit time(each minute) is called GFR. In normal adult human, GFR average 125ml /min, 180L/day.
3. Filtration fraction(FF) The ratio between glomerular filtration rate and renal plasma flow per a minute is termed FF.  FF = GFR/RPF(renal plasma flow). In normal human , FF is 19~20%
4. Glomerular effective filtration pressure (GEFP)
GEFP is the net dynamics of glomerular filtration.
    GEFP =  glomerular capillary hydrostatic pressure (GCHP) – glomerular capillary plasma colloid osmotic pressure ( GCOP) – Bowman’s capsular hydrostatic pressure
5. Autoregulation of renal blood flow (RBF)  When the arterial blood pressure is varied between 80 and 180 mmHg, the RBF, GCHP and GFR keep constant, which is called the autoregulation of RBF and GFR. 
6. Electrostatic barrier of glomerular filtration membrane.  The three layers of glomerular filtration membrane contain negatively charged glycoprotein, these negatively charged elements impede the passage of same charged plasma protein, which is called electrostatic barrier.
7. Filtration coefficient(Kf)  Kf means the quantity of filtrate per unit time and per unit GEFP
8. Filtration equilibrium  When the force opposing filtration become equal to the force driving filtration, the GEFP is zero, filtration ceases, which is called the filtration equilibrium.
9. Osmotic diuresis  When a large amount of solute are present in tubuar fluid, the increased osmotic pressure caused by the solute retards reabsorption of water and sodium, as a result, the quantity of urine increases, which is called osmotic diuresis.
10. Glomerulotubular balance  The proximal tubular always reabsorbs the 65~70% of filtrated water and Na+ in despite of the GFR increasing and decreasing, this phenomenon is called glomerulotubular balance.
11. Water diuresis  Drinking a large amount of water causes the quantity of urine to increase, which is called water diuresis.
12. renal clearance(Cx) The renal clearance of a substance is the volume of plasma that is completely cleared of this substance by kidneys per unit time.

Ⅲ. Answer following question
1. Describe the characteristics of renal blood circulation.
Answer:
(1). Double capillary bed: The renal blood circulation has two capillary bed in series, glomerular capillary and peritubular capillary. The glomerular capillary has an high hydrostatic pressure (blood pressure), which is favorable to fluid filtration. The peritubular capillary have lower hydrostatic pressure and higher plasma colloid osmotic pressure, which is favorable to fluid reabsorption.
(2). Renal blood flow (RBF) is large. RBF of two kidney in a resting, young man,is about 1200ml /min, is about 20~25% of cardiac output.
(3). RBF is not uniformly distributed. Renal cortex receives 94% of RBF. This blood flow is necessary to sustain a high glomerular filtration rate (GRF). Outer marrow receives 5% of RBF . Inner marrow receives 1% of RBF. The relative low blood flow in the marrow preserves the osmotic gradient in this region.
(4). Characteristics of regulation of RBF
1) Autoregulation: When the arterial blood pressure is varied between 80 and 180 mmHg, the RBF, GCHP and GFR keep constant, which is called the autoregulation of RBF and GFR. 
2) Nervous regulation and humoral regulation:
Sympathetic nerve: The renal sympathetic nerves innervate the renal vascular smooth muscle. In response to the state of emergency (critical condition), the increased activity of sympathetic nerve causes the renal blood vessel to constrict, RBF decreases.
Adrenaline (epinephrine ) and noradrenaline (norepinephrene ) causes the renal blood vessel to constrict, RBF decreases.
Prostaglandin E2 (PGE) , prostacyclin(PGI2), NO, bradykinin, atrial natriuretic peptide (ANP) cause renal vessel to relax, RBF increasing.
2. What is glomerular filtration membrane? What are characteristics of glomerular filtration membrane (GFM)?
Glomerular filtration membrane means the membranous structure between the glomerular capillary blood and Bowman’s capsula cavity.     
  GFM consists of three layers- capillary endothelium, basement membrane, and epithelial  cell(podocytes). They are both structural base and barrier of glomerular  filtration,
  (1). The capillary endothelial cell layer has many round, window-like holes , which is called fenestrea , its diameter is about 50~100 nm. Because the diameter of fenestrea is relatively large, the endothelial layer is not a major barrier for filtration.
(2). The basement membrane consist of meshwork of fine fibrils ( collagen and proteoglycan fibrils) embedded in a gel-like matrix. The basement membrane contains mesh holes about 2~8 nm in diameter. The mesh holes prevents effectively filtration of plasma protein, and is major barrier for plasma protein.
(3). The epithelial cell layer consist of “foot cell ” or podocytes. The epithelial cell layer contains slit pores about 4~12 nm in diameter. 
Accordingly, GFM is quite porous, has high permeability to water and low-molecules solute, but restricts passage of the plasma protein.
The three layers of GFM contain negatively charged glycoprotein, these negatively charged elements impede the passage of same charged plasma protein, which is called electrostatic barrier.
3. Describe factors determining glomerular filtration rate.
Answer:
(1). GEFP (glomerular effective filtration pressure):
1) GCHP (glomerular capillary hydrostatic pressure): The GCHP is the only driving force for filtration. GCHP is determined by the arterial blood pressure and the contractile state of afferent and efferent arteriole. .
  When the arterial blood pressure is varied between 80 and 180 mmHg, the RBF, GCHP and GFR keep constant, which is called the autoregulation of RBF and GFR. 
If arterial pressure is lowered below 80mmHg, the GCHP will markedly fall, and thus GFR will reduce. 
If arterial pressure is declined below 50mmHg, the GEFP falls to zero, GFR is zero, the patient or animal will be anuria.
  When constriction of the afferent arteriole strengthen, the precapillary resistance increase; GCHP declines, GEFP decreases; GFR decreases. When the constriction of efferent arteriole strengthen, postcapillary resistance increases; GCHP elevates; GFR increases.
2) GCOP (glomerular capillary plasma colloid osmotic pressure): The GCOP is force opposing filtration; normally, it is stable.
With intravenous infusion of a large volume of normal saline, a decreased GCOP increases the GEFP and GFR.
3) Boeman’s capsule hydrostatic pressure: Boeman’s capsule hydrostatic pressure is the force opposing filtration. Normally, Bowman’s capsule hydrostatic pressure is stable. In certain pathological state, such as obstruction of urinary tract, Bowman’s capsule pressure increases, GFR reduces. 
  (2). Renal plasma flow (RPF)
  GFR is in direct proportion to RPF.
Mechanism : As mention above, the GCOP will increase due to prior filtration. When RPF increases, the increase of GCOP along the length of glomerular capillary will become slow, the location at which the filtration equilibrium is achieved nears the efferent arteriolar end. The effective filtration membrane area increases, GFR increases. Conversely, if the RPF decreases, GFR decreases.
  (3). Filtration coefficient ( KF) 
  GFR is in direct proportion to Kf.  GFR = Kf × GEFP
Kf means the quantity of filtrate per unit time and per unit GEFP .
Kf is in direct proportion to both the fluid permeability and surface area of filtration membrane.
If there is a decrease in either permeability or surface area of filtration membrane, GFR will decrease. In chronic renal  disease, the glomeruli are destroyed, leading to a reduction in filtration membrane area and a diminished GFR. If some diseases damage the mechanical and electrostatic barrier, the permeability of filtration membrane increases, leading to hematuria and  proteinuria.
4. When a person have a large amount of haemorrhage and thus his arterial blood pressure decreases markedly, how change his quatity of urine? Why?
  Answer:
    His quantity of urine decreses.
    Mechanism:
(1) GFR decreses: The declined excessively arterial blood pressure result in the glomerular capillary hydrostatic pressure to decline, the glomerular effective filtration pressure reducing, glomerular filtration rate (GFR) decreasing.
(2) Renal plasma flow decreases: Both declined arterial blood pressure and the activity of sympathetic nerve causes the renal plasma flow to decrease; the increase of the plasma colloid osmotic pressure along the length of glomerular capillary will become quick, the location at which the filtration equilibrium is achieved nears the afferent arteriolar end. Consequently, effective filtration membrane area decreases, GFR decreases.
(3) Release of ADH increases:The stimulation of the decreased excessively blood volume to the volume receptors located on left atrium and thoracic large vines decreases; the afferent signals of the vagus nerve decreases; the release of ADH increases reflexly. The increased ADH plasma concentration elevates the permeability of distal convoluted tubule and collecting duct to water, thereby increasing water reabsorption, the quantity of urine decreasing and urine concentrates.
(4) Release of aldosterone increases: The decreased blood pressure at the afferent arteriole and the excessive activity of sympathetic nerve causes the juxtaglomerular cell to release rennin. The rennin-angiotensin-aldosterone system is started. Release of aldosterone increases. The aldosterone promotes the distal convoluted tubule and collecting duct to reabsorb Na+ and water. 
5. Describe the process of reabsorption of N+ and Cl- at proximal tubule. 
Answer:
The N+ - K+ pump located on basolateral membrane of epithelial cell of the proximal tubule actively transports Na+ from cell to interstitial fluid and K+ from interstitial fluid into cell. The operation of Na+-K+ pump carries out the reabsorption of Na+, maintains the low Na+ and the high K+ of intracellurar fluid and creates a negative potential within cell. This electrochemical gradient will provides energy for symport , antiport .
In the apical membrane of epithelial cell of the first half of proximal tubule, Na+ is reabsorbed mainly by symport along with glucose and by antiport with H+ from tubular fluid into epithelial cell;HCO3- is preferentially reabsorbed, Cl- is not reabsorbed. Therefore, the Cl- concentration increases gradually. In second half of proximal tubule, the Cl- diffuse from tubular lumen into intercellular fluid via the tight junction down concentration difference, creating a transtubular potential difference that is positive in tubular lumen and is negative in interstitial fluid, this potential difference causes Na+ to diffuse into interstitial fluid. This is passive transport..
6. Describe the process of reabsorption of N+ and Cl-  at thick segment of Henle’s loop ascending limb
    Answer:
The N+ - K+ pump located on basolateral membrane of epithelial cell of the thick segment of Henle’s loop ascending limb actively transports Na+ from cell to interstitial fluid and K+ from interstitial fluid into cell. The operation of Na+-K+ pump carries out the reabsorption of Na+, maintains the low Na+ and the high K+ of intracellurar fluid and creates a negative potential  within cell. This electrochemical gradient will provides energy for symport , antiport .
There are a symporter of Na+, K+, Cl- on apical membrane of the cell of the thick segment ascending limb; Cl-. Na+, K+, Cl- combine with the sympoter in the proportion of 1-Na+:2-Cl-:1K+, then the Na+ and Cl- moves down its electrochemical gradient, while K+ against their electrochemical gradient from tubular fluid into the cell.
The increased intracellular Na+ is actively transported by Na+-K+ pump on basolateral membrane into interstitial fluid, the Cl- diffuses passively into interstitial fluid, the K+ partly into interstitial fluid and partly diffuses back tubular fluid and thus cause the tubular fluid to be positive potential. The positive potential in tubular fluid promotes Na+, K+ and Ca2+ to be passively reabsorbed via the paracellular transport (50%). 
7. Describe the process of reabsorption of N+ and Cl-  at the distal convoluted tubule and collecting duct.
  Answer:
The Na+ - K+ pump located on basolateral membrane of the principal cell of the distal convoluted tubule and collecting duct actively transports Na+ from cell to interstitial fluid and K+ from interstitial fluid into cell. The operation of Na+-K+ pump carries out the reabsorption of Na+, maintains the low Na+ and the high K+ of intracellurar fluid and creates a negative potential within cell. This electrochemical gradient will provides energy for symport of Na+ - Cl-.
There are a symporter of Na+ - Cl- on the luminal membrane of of the initial segment of distal convoluted tubule. The Na+ and Cl- are transported across the luminal membrane from tubular fluid into cell by the symport.
  There are a Na+ channel on the luminal membrane of principal cell in the latter second half segment of distal convoluted tubule and collecting duct. The Na+ in tubular fluid diffuse into cell through the Na+ channel, and thus creates a negative potential in the tubular fluid. The negative potential drives the Cl- reabsorption via paracelluar transport and K+ secretion.
8. Describe the identical points and different points of reabsorption of N+ and Cl- at proximal tubule, thick segment of Henle’s loop ascending limb, distal convoluted tubule and collecting duct.
  Answer:
  The identical points: The N+ - K+ pump located on basolateral membrane of epithelial cell of the proximal tubule, distal tubule, and collecting duct primarily actively transports Na+ from cell to interstitial fluid and K+ from interstitial fluid into cell. The operation of Na+-K+ pump carries out the reabsorption of Na+, maintains the low Na+ and the high K+ of intracellurar fluid and creates a negative potential of about -70 mv within cell. This electrochemical gradient will provides energy for symport , antiport . Reabsorption of Cl- is passive transport.
  The different points:
① In the epithelial cell apical membrane of the first half of proximal tubule, Na+ is reabsorbed mainly by symport along with glucose and by antiport with H+ from tubular fluid into epithelial cell;HCO3- is preferentially reabsorbed, Cl- is not reabsorbed. Therefore, the Cl- concentration increases gradually. In second half of proximal tubule, the Cl- diffuse from tubular lumen into intercellular fluid via the tight junction down concentration difference, creating a transtubular potential difference that is positive in tubular lumen and is negative in interstitial fluid, this potential difference causes Na+ to diffuse into interstitial fluid. This is passive transport..
②The apical membrane of the cell of the thick segment ascending limb has a symporter of Na+, K+, Cl-. Na+, K+, Cl- combine with the sympoter in the proportion of 1-Na+:2-Cl-:1K+, then the Na+ and Cl- moves down its electrochemical gradient, while K+ against their electrochemical gradient from tubular fluid into the cell. The increased intracellular Na+ is actively transported by Na+-K+ pump on basolateral membrane into interstitial fluid, the Cl- diffuses passively into interstitial fluid, the K+ partly into interstitial fluid and partly diffuses back tubular fluid and thus cause the tubular fluid to be positive potential. The positive potential in tubular fluid promotes Na+, K+ and Ca2+ to be passively reabsorbed via the paracellular transport (50%).
③There are a symport Na+ - Cl- on the luminal membrane of of the initial segment of distal convoluted tubule. The Na+ and Cl- are transported across the luminal membrane from tubular fluid into cell by the symport.
  There are a Na+ channel on the luminal membrane of principal cell in the latter second half segment of distal convoluted tubule and collecting duct.  The Na+ in tubular fluid diffuse into cell through the Na+ channel, and thus creates a negative potential in the tubular fluid. The negative potential drives the Cl- reabsorption via paracelluar transport and K+ secretion.
9. Describe Na+-glucose symport and Na+-H+ antiport in the proximal tubule.
Answer:
The Na+ - K+ pump located on basolateral membrane of epithelial cell of the proximal tubule, distal tubule, and collecting duct transports Na+ from cell to interstitial fluid and K+ from interstitial fluid into cell. The operation of Na+-K+ pump carries out the reabsorption of Na+, maintains the low Na+ and the high K+ of intracellurar fluid and creates a negative potential  within cell. This electrochemical gradient will provides energy for symport and antiport .
In the epithelial cell luminal membrane (apical membrane) of proximal tubule, Na+ is reabsorbed by symport along with glucose and by antiport with H+, from tubular fluid into epithelial cell.  Na+-glucose symport: The Na+ and glucose of tubular fluid bind with the same co-transporter of luminal membrane, then, the Na+ moves into the cell down its electrochemical gradient while the glucose is pulled into cell against its concenteation difference. Then, this increased intracellular glucose diffuse via facilitated diffusion via carrier into interstitial fluid and blood.
Na+-H+ antiport : The intracellular H+ and Na+ in tubular lumen bind with the same antiporter of luminal membrane, then, the Na+ move into cell down its electrochemical gradient while the H+ is pushed out of cell against its electrochemical gradient into tubular lumen, which is called H+-Na+ exchange.
10.describe the process of reabsorption of water in the renal tubule and collecting duct.
Answer:
The manner of water reabsorption is osmosis, is passive transport. The direct driving force for water reabsorption is the osmolarity gradient between interstitial fluid and tubular fluid. The osmolarity (or osmotic pressuere) of the interstitial fluid is higher than that of the tubular fluid, then water penetrates down the osmolarity gradient from the tubular fluid to the interstitial fluid.  This osmolarity gradient result from the reabsorption of Na+, Cl- ,and other solutes. Therefore, reabsorption of water is directly related to the reabsorption of solute. The proximal tubule is high permeable to water. The reabsorption of water in proximal tubule is iso-osmotic reabsorption. The descending limb of Henle’s loop is high permeable to water, water diffusing from tubular fluid to medulla interstitial fluid and thus causing the osmolarity of tubular fluid to be increased. The ascending limb of Henle’s loop is impermeable to water, water can not being reabsorbed at this portion. The reabsorption of water in proximal tubule and Henle’s loop is relatively fixed and is not affected by regulatory factors.
The reabsorption of water at distal convoluted tubule and collecting duct is regulated by ADH. In the presence of high concentration ADH, the permeability of these tubule to water increases markedly, water being reabsorbed. The increased water and solutes in the intercellular fluid elevates the hydrostatic pressure in this region and thereby drives fluid into peritubular capillary.
11. Describe the process of secretion of K+ in distal convoluted and collecting duct.
Answer:
The vast majority of filtrated K+ is reabsorbed in proximal tubule and Henle’s loop. The K+ in end-urine derives mainly from secretion of principal cell in distal convoluted tubule and collecting duct.
The positions of K+ secretion are the distal convoluted tubule and collecting duct.
The manner of K+ secretion is a passive transport.
The driving force of K+ secretion is the negative potential in tubular fluid created by the active reabsorption of Na+.
The Na+-K+ pump located on basolateral membrane of principal cell of distal convoluted tubule and collecting duct actively transports Na+ from cell into interstitial fluid and causes the high K+ and low Na+ of the intracellular fluid. Na+ of tubular fluid diffuses down electrochemical gradient into cell, and thus creates a negative potential in the tubular fluid. This negative potential and increased intracellular K+ drives K+ from cell into tubular fluid. Therefore , K+ secretion is positively related to the activity of Na+-K+ pump and the permeability of luminal membrane to Na+.
12. Describe the process of H+ secretion and HCO3- reabsorption  in renal tubule, collecting duct and deacribe its sinificance.
In proximal tubule and thick segment of Henle’s loop ascending limb, H+ is secreted by Na+-H+ exchange.
Na+-H+ antiport : The intracellular H+ and Na+ in tubular lumen bind with the same antiporter of luminal membrane, then, the Na+ move into cell down its electrochemical gradient while the H+ is pushed out of cell against its electrochemical gradient into tubular lumen, which is also called H+-Na+ exchange. The H+ secretion couples with the reabsorption of HCO3-.
The HCO3-(bicarbonate) filtrated can not permeate the luminal membrane, therefore HCO3- can not be directly reabsorbed.   
In tubular fluid  Carbonic
Anhydrase of
apical membrane
,                HCO3- + H+ ←  ——— →  H2CO3 → H2O +CO2
The CO2 diffuses down its concentration difference into tubular cell .
            In tubular cell  Carbonic
anhydrase
              CO2 + H2O ←———→ H2CO3 → H+ + HCO3-
The majority of HCO3- are transported along with Na+ into interstitial fluid and then blood, while the H+ is actively secreted into tubular fluid by Na+-H+ exchange.
Therefore, the manner of HCO3- reabsorption is CO2 .
In the distal tubule and collecting duct, the intercalated cell actively secretes H+, its process is following:
There are much CO2  in the intercalated cells. These CO2 are from both the metabolism of cell and blood.
        In intercalated cell  Carbonic
anhydrase
              CO2 + H2O ←———→ H2CO3 → H+ + HCO3-     
The HCO3- is reabsorbed into interstitial fluid, while the H+ is actively transported by proton pump( H+- ATPase)and H+-K+ ATPase into tubular fluid.
Significance: ①Whenever a H+ is secreted , there must be a HCO3- is generated or reabsorbed. Kidneys maintains the acid-base balance of internal environment by reabsorbing HCO3- and secreting H+. When pH value of internal environment lowers, the activity of carbonic anhydrase increases, secretion of H+ and reabsorption of HCO3- increasing. .② It promotes NH3 secretion. H+ in tubular fluid bind with NH3 to form NH4+. NH4+concentration of tubular fluid decreasing, NH3 secretion increasing.
13. Giving intravenously hyperosmotic glucose solution to a animal (or person), how changes its (or his ) quantity of urine ? why ?
Answer:
Giving intravenously hyperosmotic glucose solution to a animal (or person) leads to the osmotic diuresis. The quantity of urine increases.
Mechanesm: When the hyperosmotic glucose solution is injected intravenously, the blood glucose concentration rises excessively and is higher than the renal glucose threshold. The glucose can not be reabsorbed completely. The excessive glucose remains in the tubular fluid, the osmotic pressure of tubular fluid increases, which retards water reabsorption, the quantity of urine  increasing. For the same reason, the quantity of urine of the most of parients with diabetes increases(hyperuria).
14. When a person is subjected a large amount of sweating and deprivation of drinking water, how change his quantity of urine? Way?
Answer:
His quantity of urine decreases.
Mechnism: The reasons for this are: (1) His plasma crystal osmotic pressure increases. The increased  plasma crystal osmotic pressure stimulates the osmoreceptor in hypothalamus and thus causes posterior pituitary to release ADH. The increased ADH plasma concentration elevates the permeability of distal convoluted tubule and collecting duct to water, thereby increasing water reabsorption, the quantity of urine decreasing and urine concentrates.
(2) His blood volume decrease. The stimulation of the decreased blood volume to the volume receptors located on left atrium and thoracic large vines (such as vena cava, pulmunary vines) decreases; the afferent signals of the vagus nerve decreases; the release of ADH increases reflexly. Finally, the quantity of urine decreases for the same reason.
Conversely, when a person drinks a large amount of water, the opposite sequence of events occurs.
15. When a person drinks a large amount of water, how change his quantity of urine? Way?
  Answer:
  His quantity of urine increases
  Mechanism: (1) After drinking a large amount of water, his plasma is diluted. The stimulation of decreased plasma crystal osmotic pressure to the osmoreceptor in hypothalamus weaken and thus causes the release of ADH to reduce. The decreased ADH plasma concentration reduces the permeability of distal convoluted tubule and collecting duct to water. The reabsorption of water decreases, the quantity of urine increasing.
  (2) After drinking a large amount of water, his blood volume increases. The increased blood volume stimulates the volume receptors located on left atrium and thoracic large vines, the afferent signals of the vagus nerve increasing, the release of ADH reducing reflexly. For the same reason, the quantity of urine increases.   
16. Describe the function and its functional mechanisms of aldosterone on renal tubule and collecting duct
The aldosterone promotes the Na+ reabsorption and K+ secretion in distal convoluted tubule and collecting duct.
Mechanism: Aldosterones enter the principal cells; bind with the receptor in cytoplasm to form a hormone-cytoplasm receptor complex. The complex are transported into the nucleus, then bind with the receptor in the nucleus to form hormone-nucleus receptor complex and thus activates the transcription(转录)process of specific genes to form specific mRNA. The mRNA diffuses into cytoplasm, where it promotes the translation(转化)process to form aldosterone-induced proteins. The latter ① increase the number of the Na+ channels and thus increase the Na+ permeability of the luminal membrane , ② enhance the activity of the Na+ - K+ ATPase pump on the basolateral membrane, ③ increase the synthesis of ATP.
17. Describe the mechanism by which the renal medullary osmolarity gradient formed.
(1). Mechanism forming osmolarity gradient in outer medulla
In outer medulla, the increased osmolarity of interstitial fluid result mainly from the active reabsorption of Na+ and Cl-, while water can not be reabsorbed in the ascending limb thick segment of Henle’s loop. The ascending limb thick segment of Henle’ s loop is impermeable to water, actively reabsorbs Na+ and Cl-, consequently, the NaCl concentration in interstitium is increased, rising osmolarity of interstitial fluid in outer medulla .
(2) . Mechanism forming osmolarity gradient in inner medulla
In inner medulla, the increased concentration of NaCl and urea contribute equally to the increased osmolarity of interstitial fluid .
The process forming the osmolarity gradient of inner medulla is countercurrent multiplication,  is following:
1)The descending limb thin segment of of Henle’s loop is impermeable to urea and NaCl, is highly permeable to water. As tubular fluid flow through this region, water is net reabsorbed, the concentration of NaCl in tubular fluid progressively increases.
2)When this tubular fluid rounds the bend of Henle’s loop and flow into ascending limb thin segment, because the ascending limb thin segment is highly permeable to NaCl and is impermeable to water, NaCl diffuses down concentration difference from tubular fluid into inner medullary peritubular fluid, thereby increasing its osmolarity.
3) Urea circulation increases the urea concentration of inner medulla
The distal convoluted tubule and cortical segment and outer medullary segment of collecting duct is impermeable to urea. In the presence of high concentration ADH, water is reabsorbed from these segments, increasing urea concentration in tubular fluid.
Because the inner medullary collecting duct is highly permeable to urea, then urea diffuses down its concentration difference from tubule into peritubular interstitium and thus increase the urea concentration and osmolarity of peritubular fluid in the inner medulla.
The urea in inner medulla diffuses into ascending limb thin segment, then flow through ascending limb thick segment, distal convoluted tubule, outer medullary segment of collecting duct , and finally back to the inner medullary collecting duct where urea diffuses into interstitial fluid again, this process is called urea circulation .
18. Describe the process of urine dilution and concentration.
(1) Urine dilution
In the ascending limb thick segment of Henle’s loop, Na+, K+, Cl- are avidly reabsorbed, however, this portion of tubule is impermeable to water, therefore the tubular fluid become dilute , its osmolarity is about 100mmol/L. As these dilute fluid flows through the distal tubule and collecting duct, the Na+, K+,Cl- is continuously reabsorbed. In the absence of ADH, these tubule are also impermeable to water. Therefore, the tubular fluid become even more dilute, its osmolarity lowers as low as 50 mOsm/L.
(2) Urine concentration.
The urine concentration occurs in the medullary segment of collecting duct. In presence of high concentration ADH, the distal convoluted tubule and collecting duct is high permeable to water. When tubular fluid flow through the medallary segment of collecting duct, because there is a osmolarity gradient in the interstitial fluid of renal medalla, water moves down osmotic pressure difference from tubular fluid into renal medullary interstitial fluid, up to osmotic equilibrium is achieved, consequently, the quantity of urine decreases and urine osmolarity increases.
离线孙笑天
发帖
97
啄木币
1729
鲜花
82
只看该作者 3楼 发表于: 2007-12-15
                          英文题  第八章 (2006. 3. 23)
Ⅰ. 选择题
1. The main difference between primary urine and blood plasma is:  D
  A. Glucose concentration.
B. Crystal osmotic pressure.
C. NaCl concentration.
D. Plasma protein concentration.
E. pH value.
2. Site of renin secretion is at  A
  A. Juxtaglumerular cells.
B. Extraglomerular mesangial cells.
C. Juxtaglomerular apparatus.
D. Macula densa.
E. Glomerular capillary
3. Macula densa is a  A
A. Chemical receptor.
B. Baroreceptor.
C. Stretch receptor.
D. Volume receptor.
E. Osmoreceptor.
4. Characteristic of renal blood circulation is:  A
  A. Double capillary beds.
B. Low capillary hydrostatic pressure.
C. High peritubular capillary hydrostatic pressure.
D. Smaller blood flow of renal cortex.
E. Greater blood flow of renal marrow (medulla).
5. Which of the following descriptions about urine of a normal adult is true ? D
  A. Daily urine quantity is 2 to 3 L/ day.
B. Its osmotic pressure is equal to that of plasma.
C. It contains a large amount of NaCl.
D. Urine does not contain glucose.
E. Urine contains protein.
6. Which of the followings can increase the glomerular filtration rate ? C
  A. Arterial blood pressure rises from 80 to 180 mmHg.
B. Arterial blood pressure declines from 180 to 80 mmHg.
C. A large amount of normal saline is given intravenously.
D. Excitation of sympathetic nerve.
E. Constriction of afferent arteriole.
7. The driving force for glumerular filtration is:  A
  A. Glomerular capillary hydrostatic pressure.
B. Arterial blood pressure.
C. Plasma colloid osmotic pressure.
D. Bowman’s capsular hydrostatic pressure.
E. Blood pressure of efferent arteriole.
8. The main barrier of glomerular filtration membrane is located on B
  A. Capillary endothelial cell layer.
B. Basement membrane.
C. Visceral epithelial cell (“foot cell ” or podocyte) layer.
D. Parietal epithelial cell layer.
E. Negatively charged elements of capillary endothelial cell layer.
9. If some diseases damage the negatively charged elements of glomerular membrane, which of the following will occur ? B
  A. Glomerular filtration rate increases.
B. Proteinuria (protein in urine).
C. Hematuria ( hematuresis or blood in the urine).
D. Renal blood flow increases.
E. Plasma protein concentration increases.
10. Glucose concentration of the ultrafiltrate (primary urine) is: A
  A. Similar to blood plasma.
B. Higher than that of plasma.
C. Lower than that of plasma.
D. Similar to urine.
E. Similar to tubular fluid in distal tubule.
11. Normally, which of following substances can not move through (pass across or penetrate) glomerular filtration membrane? A
  A. Plasma protein 
B. Glocose
C. Creatinin
D. urea
E. Inulin
12. When noradrenaline (norepinephrine ) is given intravenously, the main cause of urine quantity decreasing is: B
  A. Boeman’s capsule hydrostatic pressure increases.
B. Renal arteriole constricts. 
C. Release of ADH increases.
D. Release of renin decreases.
E. Release of aldosterone decrease.   
13. Which of the following descriptions about glomerular filtration membrane is wrong?  E
  A. It consists of capillary endothelium, basement membrane, and epithelial cell.
B. It is quite porous.
C. It is structural base of glomerular filtration.
D. The basement membrane is major barrier for plasma protein.
E. The negatively charged protein move easily through the filtration membrane.
14. If reabsorption of renal tubule and collecting duct to water decreases 1%, the quantity of urine will increase  D
  A. 1%
B. 2%
C. 50%
D. 100%
E. 300%
15.Which of the following descriptions about the reabsorption of Na+ and Cl- at the proximal
tubule is wrong?  E
A. The intracellular Na+ are transported by Na+ - K+ pump to interstitial fluid   
B. The process by which Na+ move from tubular fluid into cell is passive transport. 
C. In second half of proximal tubule, Na+ and Cl-diffuse from tubular lumen into intercellular fluid mainly via the tight junction.
D. The first half of proximal tubule does not reabsorb Cl- .
E. It is regulated by aldosterone
16. Proximal tubule of kidneys reabsorbs  C
  A. 85% of filtrated water.
B. 85% of filtrated Na+.
C. 65~70% of filtrated Na+ and water.
D. 65~70% of filtrated glucose.
E. 65~70% of filtrated HCO3-.
17. Location reabsorbing glucose is at: A
  A. Proximal tubule.
B. Descending limb of Henle’ loop.
C. Ascending limb of Henle’ loop.
D. Distal convoluted tubule.
E. Collecting duct.
18. Normally, proximal tubule reabsorbs:  E
  A. 20% of filtrated glucose.
B. 50% of filtrated glucose.
C. 65~70% of filtrated glucose.
D. 80% of filtrated glucose.
E. 100% of filtrated glucose.
19. The reason for that the filtration occurs only in afferent
arteriole end of the glomerular capillary is:  B
  A. Blood pressure at the efferent arteriole end of glomerular capillary is low
B. Plasma colloid osmotic pressure gradually increases along the glomerular capillary
C. Plasma crystal osmotic pressure gradually decreases along the glomerular capillary
D. Plasma crystal osmotic pressure gradually increases along the glomerular capillary
E. Hydrostatic pressure in Bowman’s capsular gradually increases
20. Key dynamic of the solute and water reabsorption at the proximal tubule is: A
  A. Na+-K+ pump
B. Carbonic anhydrase
C. Concentration gradient
D. Potential gradient
E. Osmotic pressure gradient
20. Site that is high permeable to water and impermeable to Na+ and Cl- is C
  A. Ascending limb thick segment of Henle’ s loop
B. Ascending limb thin segment of Henle’ s loop
C. Descending limb thin segment of of Henle’s loop
D. Descending limb thick segment of of Henle’s loop
E. Distal convoluted tubule
21. Site that can actively reabsorb Na+, Cl- and K+ but is impermeable to water is:  A
A. Ascending limb thick segment of Henle’ s loop
B. Ascending limb thin segment of Henle’ s loop
C. Descending limb thin segment of of Henle’s loop
D. Descending limb thick segment of of Henle’s loop
E. Collecting duct
22. Site that is high permeable to urea is:  D
  A. Distal convoluted tubule
B. Cortical segment of collecting duct
C. Outer medullary segment of collecting duct
D. Inner medullary segment of collecting duct
E. Descending limb thin segment of of Henle’s loop
23. Cause of polyuria (quantity of urine increases) in patient with diabetes mellitus mainly is: C
  A. Glumerular filtration rate increases
B. Colloid osmotic pressure of plasma decreases
C. Osmotic pressure of tubular fluid increases
D. Crystal osmotic pressure of plasma increases
E. Secretion of ADH decreases
24. Location in which Na+ is passively reabsorbed is: B
A. The first half of proximal tubule
B. The second half of proximal tubule
C. Ascending limb thick segment of Henle’ s loop
D. Distal convoluted tubule
E. Collecting duct
25. Mechanism forming osmolarity gradient in the outer medulla mainly is A
  A. NaCl reabsorption in the ascending limb thick segment of Henle’ s loop
B. Urea circulation
C. NaCl reabsorption in the descending limb thin segment of of Henle’s loop
D. NaCl reabsorption in the descending limb thick segment of of Henle’s loop
E. Urea reasorption in the ascending limb thick segment of Henle’ s loop
26. Which of the following descriptions about H+ secretion is wrong ? D
  A. Proximal tubule secretes H+ by H+-Na+ exchange (antiport of H+ and K+)
B. Intercalated cell in distal tubule and collecting duct secretes H+ via proton pump( H+- ATPase) 
C. Intercalated cell distal tubule and collecting duct secretes H+ via H+-K+ ATPase
D. When pH value of internal environment lowers, the activity of carbonic anhydrase decreases
E. H+ secretion promotes the reabsorption of HCO3- and the secretion of NH3
27. Which of the following descriptions about NH3 secretion is wrong  D
  A. NH3 derives mainly from the deamination of glutamine
B. NH3 is a lipid-soluble weak base
C. NH3 diffuses readily from epithelial cell into tubular fluid
D. NH3 secretion inhibits H+ secretion
E. NH3 secretion promote the reabsorption of HCO3-
28. Which of the following description about K+ secretion is wrong D
A. K+ in end-urine derives mainly from the secretion of distal convoluted tubule and collecting duct.
B. Principal cells secrete K+
C. Aldosterone promotes K+ secretion
D. Antidiuretic hormone inhibit K+ secretion
E. K+ secretion is a passive transport
29. Location in which the glomerulotubular balance occurs is:  A
  A. Proximal tubule.
B. Descending limb of Henle’s loop.
C. Ascending limb of Henle’s loop.
D. Henle’s loop.
E. Distal convoluted tubule and collecting duct.
30. Which of the following factors does not concerned with the osmotic diuresis E
  A. Increased solute concentration of tubular fluid.
B. Hyperosmotic glucose solution is given intravenously.
C. Mannitol is given intravenously.
D. Diabetes mellitus
E. Diabetes insipidus
31. Location in which water is regulatory reabsorbed is in:  E
  A. Proximal tubule
B. Descending limb of Henle’s loop
C. Ascending limb of Henle’s loop
D. Henle’s loop
E. Distal convoluted tubule and collecting duct
32. Normally, hormone regulating the reabsorption of water is:  D
  Normally, hormone maintaining the water balance of body is  D
A. Renin
B. Adrenaline (epinephrine)
C. Angiotensin Ⅱ
D. ADH ( antidiuretic hormone)
E. Aldosterone
33. Normally, the Na+ reabsorption and K+ secretion are regulated by: E
  Normally, hormone maintaining the Na+ and K+ balance is: E
A. Renin
B. Adrenaline (epinephrine)
C. Angiotensin Ⅱ
D. ADH ( antidiuretic hormone)
E. Aldosterone
34. After drinking a large amount of water, the reason for increased quantity of urine is that: C
  A. Glomerular filtration rate increases
B.. Aldosterone secretion decreases
C. ADH secretion decreases
D. Arterial blood pressure rises
E. Plasma colloid osmotic pressure decreases
35. After a large amount of sweating, the cause resulting in the decreased quantity of urine is that: C
A. Plasma crystal osmotic pressure declines
B. Plasma colloid osmotic pressure elevates
C. ADH secretion increases
D. Aldosterone secretion increases
E. Renal blood flow decreases
36. Main reason for that excitation of sympathetic nerve results in decreasing of urine quantity is: A
A. Renal blood flow decreases
B. Plasma colloid osmotic pressure elevate
C. ADH secretion increases
D. Aldosterone secretion decreases
E. Glomerular filtration rate increases
37. Location in which urine concentration occurs is:  D
A. Ascending limb thick segment of Henle’s loop
B. Distal convoluted tubule
C. Cortical segment of collecting duct
D. Medullary segment of collecting duct
E. Descending limb thin segment of Henle’s loop
38. Which of followings is not related to (concerned with) the urine dilution E
A. Ascending limb thick segment of Henle’ s loop
B. Distal convoluted tubule
C. Cortical segment of collecting duct
D. Inner medullary segment of collecting duct
E. Proximal tubule
39. Which of followings does not initiate ADH secretion ? B
  A. Dehydration (deficit of water)
B. Decreased plasma colloid osmotic pressure
C. Increased plasma crystal osmotic pressure
D. Severe pain
E. A large mount of hemorrhage 
40. Which of the followings does not initiate renin secretion ? E
  A. Reduction of afferent arteriolar transmural pressure
B. Reduction of NaCl flowing through the macula densa
C. Increased sympathetic nerve activity
D. Elevated plasma adrenaline and noradrenaline
E. Decreased intrarenal prostaglandin E2 formation.
41. Which of the followings does not cause aldosterone secretion ? D
  A. Decreased arterial blood pressure
B. Angiotensin Ⅱ
C. Angiotensin Ⅲ
D. Increased plasma Na+ concentration
E. Increased plasma K+ concentration
42 Normally, which of the following substances has the minimal renal clearance? A
  A. Glucose
B. Urea
C. Endogenous creatinine
D. Inulin
E. NaCl
Ⅱ. Define following word
1. Glomerular filtration When blood flows through the glomerular capillary, under driving of glomerular effective filtration pressure, the water and low-molecular weight solute in plasma move through filtration membrane into the Bowman’s capsule, this process is termed as glomerular filtration. The filtrate formed in glomerulus is ultrafiltrate, is also called the primary urine.
2. Glomerular filtration rate (GFR)  The quantity of ultrafiltrate formed by both kidneys per unit time(each minute) is called GFR. In normal adult human, GFR average 125ml /min, 180L/day.
3. Filtration fraction(FF) The ratio between glomerular filtration rate and renal plasma flow per a minute is termed FF.  FF = GFR/RPF(renal plasma flow). In normal human , FF is 19~20%
4. Glomerular effective filtration pressure (GEFP)
GEFP is the net dynamics of glomerular filtration.
    GEFP =  glomerular capillary hydrostatic pressure (GCHP) – glomerular capillary plasma colloid osmotic pressure ( GCOP) – Bowman’s capsular hydrostatic pressure
5. Autoregulation of renal blood flow (RBF)  When the arterial blood pressure is varied between 80 and 180 mmHg, the RBF, GCHP and GFR keep constant, which is called the autoregulation of RBF and GFR. 
6. Electrostatic barrier of glomerular filtration membrane.  The three layers of glomerular filtration membrane contain negatively charged glycoprotein, these negatively charged elements impede the passage of same charged plasma protein, which is called electrostatic barrier.
7. Filtration coefficient(Kf)  Kf means the quantity of filtrate per unit time and per unit GEFP
8. Filtration equilibrium  When the force opposing filtration become equal to the force driving filtration, the GEFP is zero, filtration ceases, which is called the filtration equilibrium.
9. Osmotic diuresis  When a large amount of solute are present in tubuar fluid, the increased osmotic pressure caused by the solute retards reabsorption of water and sodium, as a result, the quantity of urine increases, which is called osmotic diuresis.
10. Glomerulotubular balance  The proximal tubular always reabsorbs the 65~70% of filtrated water and Na+ in despite of the GFR increasing and decreasing, this phenomenon is called glomerulotubular balance.
11. Water diuresis  Drinking a large amount of water causes the quantity of urine to increase, which is called water diuresis.
12. renal clearance(Cx) The renal clearance of a substance is the volume of plasma that is completely cleared of this substance by kidneys per unit time.

Ⅲ. Answer following question
1. Describe the characteristics of renal blood circulation.
Answer:
(1). Double capillary bed: The renal blood circulation has two capillary bed in series, glomerular capillary and peritubular capillary. The glomerular capillary has an high hydrostatic pressure (blood pressure), which is favorable to fluid filtration. The peritubular capillary have lower hydrostatic pressure and higher plasma colloid osmotic pressure, which is favorable to fluid reabsorption.
(2). Renal blood flow (RBF) is large. RBF of two kidney in a resting, young man,is about 1200ml /min, is about 20~25% of cardiac output.
(3). RBF is not uniformly distributed. Renal cortex receives 94% of RBF. This blood flow is necessary to sustain a high glomerular filtration rate (GRF). Outer marrow receives 5% of RBF . Inner marrow receives 1% of RBF. The relative low blood flow in the marrow preserves the osmotic gradient in this region.
(4). Characteristics of regulation of RBF
1) Autoregulation: When the arterial blood pressure is varied between 80 and 180 mmHg, the RBF, GCHP and GFR keep constant, which is called the autoregulation of RBF and GFR. 
2) Nervous regulation and humoral regulation:
Sympathetic nerve: The renal sympathetic nerves innervate the renal vascular smooth muscle. In response to the state of emergency (critical condition), the increased activity of sympathetic nerve causes the renal blood vessel to constrict, RBF decreases.
Adrenaline (epinephrine ) and noradrenaline (norepinephrene ) causes the renal blood vessel to constrict, RBF decreases.
Prostaglandin E2 (PGE) , prostacyclin(PGI2), NO, bradykinin, atrial natriuretic peptide (ANP) cause renal vessel to relax, RBF increasing.
2. What is glomerular filtration membrane? What are characteristics of glomerular filtration membrane (GFM)?
Glomerular filtration membrane means the membranous structure between the glomerular capillary blood and Bowman’s capsula cavity.     
  GFM consists of three layers- capillary endothelium, basement membrane, and epithelial  cell(podocytes). They are both structural base and barrier of glomerular  filtration,
  (1). The capillary endothelial cell layer has many round, window-like holes , which is called fenestrea , its diameter is about 50~100 nm. Because the diameter of fenestrea is relatively large, the endothelial layer is not a major barrier for filtration.
(2). The basement membrane consist of meshwork of fine fibrils ( collagen and proteoglycan fibrils) embedded in a gel-like matrix. The basement membrane contains mesh holes about 2~8 nm in diameter. The mesh holes prevents effectively filtration of plasma protein, and is major barrier for plasma protein.
(3). The epithelial cell layer consist of “foot cell ” or podocytes. The epithelial cell layer contains slit pores about 4~12 nm in diameter. 
Accordingly, GFM is quite porous, has high permeability to water and low-molecules solute, but restricts passage of the plasma protein.
The three layers of GFM contain negatively charged glycoprotein, these negatively charged elements impede the passage of same charged plasma protein, which is called electrostatic barrier.
3. Describe factors determining glomerular filtration rate.
Answer:
(1). GEFP (glomerular effective filtration pressure):
1) GCHP (glomerular capillary hydrostatic pressure): The GCHP is the only driving force for filtration. GCHP is determined by the arterial blood pressure and the contractile state of afferent and efferent arteriole. .
  When the arterial blood pressure is varied between 80 and 180 mmHg, the RBF, GCHP and GFR keep constant, which is called the autoregulation of RBF and GFR. 
If arterial pressure is lowered below 80mmHg, the GCHP will markedly fall, and thus GFR will reduce. 
If arterial pressure is declined below 50mmHg, the GEFP falls to zero, GFR is zero, the patient or animal will be anuria.
  When constriction of the afferent arteriole strengthen, the precapillary resistance increase; GCHP declines, GEFP decreases; GFR decreases. When the constriction of efferent arteriole strengthen, postcapillary resistance increases; GCHP elevates; GFR increases.
2) GCOP (glomerular capillary plasma colloid osmotic pressure): The GCOP is force opposing filtration; normally, it is stable.
With intravenous infusion of a large volume of normal saline, a decreased GCOP increases the GEFP and GFR.
3) Boeman’s capsule hydrostatic pressure: Boeman’s capsule hydrostatic pressure is the force opposing filtration. Normally, Bowman’s capsule hydrostatic pressure is stable. In certain pathological state, such as obstruction of urinary tract, Bowman’s capsule pressure increases, GFR reduces. 
  (2). Renal plasma flow (RPF)
  GFR is in direct proportion to RPF.
Mechanism : As mention above, the GCOP will increase due to prior filtration. When RPF increases, the increase of GCOP along the length of glomerular capillary will become slow, the location at which the filtration equilibrium is achieved nears the efferent arteriolar end. The effective filtration membrane area increases, GFR increases. Conversely, if the RPF decreases, GFR decreases.
  (3). Filtration coefficient ( KF) 
  GFR is in direct proportion to Kf.  GFR = Kf × GEFP
Kf means the quantity of filtrate per unit time and per unit GEFP .
Kf is in direct proportion to both the fluid permeability and surface area of filtration membrane.
If there is a decrease in either permeability or surface area of filtration membrane, GFR will decrease. In chronic renal  disease, the glomeruli are destroyed, leading to a reduction in filtration membrane area and a diminished GFR. If some diseases damage the mechanical and electrostatic barrier, the permeability of filtration membrane increases, leading to hematuria and  proteinuria.
4. When a person have a large amount of haemorrhage and thus his arterial blood pressure decreases markedly, how change his quatity of urine? Why?
  Answer:
    His quantity of urine decreses.
    Mechanism:
(1) GFR decreses: The declined excessively arterial blood pressure result in the glomerular capillary hydrostatic pressure to decline, the glomerular effective filtration pressure reducing, glomerular filtration rate (GFR) decreasing.
(2) Renal plasma flow decreases: Both declined arterial blood pressure and the activity of sympathetic nerve causes the renal plasma flow to decrease; the increase of the plasma colloid osmotic pressure along the length of glomerular capillary will become quick, the location at which the filtration equilibrium is achieved nears the afferent arteriolar end. Consequently, effective filtration membrane area decreases, GFR decreases.
(3) Release of ADH increases:The stimulation of the decreased excessively blood volume to the volume receptors located on left atrium and thoracic large vines decreases; the afferent signals of the vagus nerve decreases; the release of ADH increases reflexly. The increased ADH plasma concentration elevates the permeability of distal convoluted tubule and collecting duct to water, thereby increasing water reabsorption, the quantity of urine decreasing and urine concentrates.
(4) Release of aldosterone increases: The decreased blood pressure at the afferent arteriole and the excessive activity of sympathetic nerve causes the juxtaglomerular cell to release rennin. The rennin-angiotensin-aldosterone system is started. Release of aldosterone increases. The aldosterone promotes the distal convoluted tubule and collecting duct to reabsorb Na+ and water. 
5. Describe the process of reabsorption of N+ and Cl- at proximal tubule. 
Answer:
The N+ - K+ pump located on basolateral membrane of epithelial cell of the proximal tubule actively transports Na+ from cell to interstitial fluid and K+ from interstitial fluid into cell. The operation of Na+-K+ pump carries out the reabsorption of Na+, maintains the low Na+ and the high K+ of intracellurar fluid and creates a negative potential within cell. This electrochemical gradient will provides energy for symport , antiport .
In the apical membrane of epithelial cell of the first half of proximal tubule, Na+ is reabsorbed mainly by symport along with glucose and by antiport with H+ from tubular fluid into epithelial cell;HCO3- is preferentially reabsorbed, Cl- is not reabsorbed. Therefore, the Cl- concentration increases gradually. In second half of proximal tubule, the Cl- diffuse from tubular lumen into intercellular fluid via the tight junction down concentration difference, creating a transtubular potential difference that is positive in tubular lumen and is negative in interstitial fluid, this potential difference causes Na+ to diffuse into interstitial fluid. This is passive transport..
6. Describe the process of reabsorption of N+ and Cl-  at thick segment of Henle’s loop ascending limb
    Answer:
The N+ - K+ pump located on basolateral membrane of epithelial cell of the thick segment of Henle’s loop ascending limb actively transports Na+ from cell to interstitial fluid and K+ from interstitial fluid into cell. The operation of Na+-K+ pump carries out the reabsorption of Na+, maintains the low Na+ and the high K+ of intracellurar fluid and creates a negative potential  within cell. This electrochemical gradient will provides energy for symport , antiport .
There are a symporter of Na+, K+, Cl- on apical membrane of the cell of the thick segment ascending limb; Cl-. Na+, K+, Cl- combine with the sympoter in the proportion of 1-Na+:2-Cl-:1K+, then the Na+ and Cl- moves down its electrochemical gradient, while K+ against their electrochemical gradient from tubular fluid into the cell.
The increased intracellular Na+ is actively transported by Na+-K+ pump on basolateral membrane into interstitial fluid, the Cl- diffuses passively into interstitial fluid, the K+ partly into interstitial fluid and partly diffuses back tubular fluid and thus cause the tubular fluid to be positive potential. The positive potential in tubular fluid promotes Na+, K+ and Ca2+ to be passively reabsorbed via the paracellular transport (50%). 
7. Describe the process of reabsorption of N+ and Cl-  at the distal convoluted tubule and collecting duct.
  Answer:
The Na+ - K+ pump located on basolateral membrane of the principal cell of the distal convoluted tubule and collecting duct actively transports Na+ from cell to interstitial fluid and K+ from interstitial fluid into cell. The operation of Na+-K+ pump carries out the reabsorption of Na+, maintains the low Na+ and the high K+ of intracellurar fluid and creates a negative potential within cell. This electrochemical gradient will provides energy for symport of Na+ - Cl-.
There are a symporter of Na+ - Cl- on the luminal membrane of of the initial segment of distal convoluted tubule. The Na+ and Cl- are transported across the luminal membrane from tubular fluid into cell by the symport.
  There are a Na+ channel on the luminal membrane of principal cell in the latter second half segment of distal convoluted tubule and collecting duct. The Na+ in tubular fluid diffuse into cell through the Na+ channel, and thus creates a negative potential in the tubular fluid. The negative potential drives the Cl- reabsorption via paracelluar transport and K+ secretion.
8. Describe the identical points and different points of reabsorption of N+ and Cl- at proximal tubule, thick segment of Henle’s loop ascending limb, distal convoluted tubule and collecting duct.
  Answer:
  The identical points: The N+ - K+ pump located on basolateral membrane of epithelial cell of the proximal tubule, distal tubule, and collecting duct primarily actively transports Na+ from cell to interstitial fluid and K+ from interstitial fluid into cell. The operation of Na+-K+ pump carries out the reabsorption of Na+, maintains the low Na+ and the high K+ of intracellurar fluid and creates a negative potential of about -70 mv within cell. This electrochemical gradient will provides energy for symport , antiport . Reabsorption of Cl- is passive transport.
  The different points:
① In the epithelial cell apical membrane of the first half of proximal tubule, Na+ is reabsorbed mainly by symport along with glucose and by antiport with H+ from tubular fluid into epithelial cell;HCO3- is preferentially reabsorbed, Cl- is not reabsorbed. Therefore, the Cl- concentration increases gradually. In second half of proximal tubule, the Cl- diffuse from tubular lumen into intercellular fluid via the tight junction down concentration difference, creating a transtubular potential difference that is positive in tubular lumen and is negative in interstitial fluid, this potential difference causes Na+ to diffuse into interstitial fluid. This is passive transport..
②The apical membrane of the cell of the thick segment ascending limb has a symporter of Na+, K+, Cl-. Na+, K+, Cl- combine with the sympoter in the proportion of 1-Na+:2-Cl-:1K+, then the Na+ and Cl- moves down its electrochemical gradient, while K+ against their electrochemical gradient from tubular fluid into the cell. The increased intracellular Na+ is actively transported by Na+-K+ pump on basolateral membrane into interstitial fluid, the Cl- diffuses passively into interstitial fluid, the K+ partly into interstitial fluid and partly diffuses back tubular fluid and thus cause the tubular fluid to be positive potential. The positive potential in tubular fluid promotes Na+, K+ and Ca2+ to be passively reabsorbed via the paracellular transport (50%).
③There are a symport Na+ - Cl- on the luminal membrane of of the initial segment of distal convoluted tubule. The Na+ and Cl- are transported across the luminal membrane from tubular fluid into cell by the symport.
  There are a Na+ channel on the luminal membrane of principal cell in the latter second half segment of distal convoluted tubule and collecting duct.  The Na+ in tubular fluid diffuse into cell through the Na+ channel, and thus creates a negative potential in the tubular fluid. The negative potential drives the Cl- reabsorption via paracelluar transport and K+ secretion.
9. Describe Na+-glucose symport and Na+-H+ antiport in the proximal tubule.
Answer:
The Na+ - K+ pump located on basolateral membrane of epithelial cell of the proximal tubule, distal tubule, and collecting duct transports Na+ from cell to interstitial fluid and K+ from interstitial fluid into cell. The operation of Na+-K+ pump carries out the reabsorption of Na+, maintains the low Na+ and the high K+ of intracellurar fluid and creates a negative potential  within cell. This electrochemical gradient will provides energy for symport and antiport .
In the epithelial cell luminal membrane (apical membrane) of proximal tubule, Na+ is reabsorbed by symport along with glucose and by antiport with H+, from tubular fluid into epithelial cell.  Na+-glucose symport: The Na+ and glucose of tubular fluid bind with the same co-transporter of luminal membrane, then, the Na+ moves into the cell down its electrochemical gradient while the glucose is pulled into cell against its concenteation difference. Then, this increased intracellular glucose diffuse via facilitated diffusion via carrier into interstitial fluid and blood.
Na+-H+ antiport : The intracellular H+ and Na+ in tubular lumen bind with the same antiporter of luminal membrane, then, the Na+ move into cell down its electrochemical gradient while the H+ is pushed out of cell against its electrochemical gradient into tubular lumen, which is called H+-Na+ exchange.
10.describe the process of reabsorption of water in the renal tubule and collecting duct.
Answer:
The manner of water reabsorption is osmosis, is passive transport. The direct driving force for water reabsorption is the osmolarity gradient between interstitial fluid and tubular fluid. The osmolarity (or osmotic pressuere) of the interstitial fluid is higher than that of the tubular fluid, then water penetrates down the osmolarity gradient from the tubular fluid to the interstitial fluid.  This osmolarity gradient result from the reabsorption of Na+, Cl- ,and other solutes. Therefore, reabsorption of water is directly related to the reabsorption of solute. The proximal tubule is high permeable to water. The reabsorption of water in proximal tubule is iso-osmotic reabsorption. The descending limb of Henle’s loop is high permeable to water, water diffusing from tubular fluid to medulla interstitial fluid and thus causing the osmolarity of tubular fluid to be increased. The ascending limb of Henle’s loop is impermeable to water, water can not being reabsorbed at this portion. The reabsorption of water in proximal tubule and Henle’s loop is relatively fixed and is not affected by regulatory factors.
The reabsorption of water at distal convoluted tubule and collecting duct is regulated by ADH. In the presence of high concentration ADH, the permeability of these tubule to water increases markedly, water being reabsorbed. The increased water and solutes in the intercellular fluid elevates the hydrostatic pressure in this region and thereby drives fluid into peritubular capillary.
11. Describe the process of secretion of K+ in distal convoluted and collecting duct.
Answer:
The vast majority of filtrated K+ is reabsorbed in proximal tubule and Henle’s loop. The K+ in end-urine derives mainly from secretion of principal cell in distal convoluted tubule and collecting duct.
The positions of K+ secretion are the distal convoluted tubule and collecting duct.
The manner of K+ secretion is a passive transport.
The driving force of K+ secretion is the negative potential in tubular fluid created by the active reabsorption of Na+.
The Na+-K+ pump located on basolateral membrane of principal cell of distal convoluted tubule and collecting duct actively transports Na+ from cell into interstitial fluid and causes the high K+ and low Na+ of the intracellular fluid. Na+ of tubular fluid diffuses down electrochemical gradient into cell, and thus creates a negative potential in the tubular fluid. This negative potential and increased intracellular K+ drives K+ from cell into tubular fluid. Therefore , K+ secretion is positively related to the activity of Na+-K+ pump and the permeability of luminal membrane to Na+.
12. Describe the process of H+ secretion and HCO3- reabsorption  in renal tubule, collecting duct and deacribe its sinificance.
In proximal tubule and thick segment of Henle’s loop ascending limb, H+ is secreted by Na+-H+ exchange.
Na+-H+ antiport : The intracellular H+ and Na+ in tubular lumen bind with the same antiporter of luminal membrane, then, the Na+ move into cell down its electrochemical gradient while the H+ is pushed out of cell against its electrochemical gradient into tubular lumen, which is also called H+-Na+ exchange. The H+ secretion couples with the reabsorption of HCO3-.
The HCO3-(bicarbonate) filtrated can not permeate the luminal membrane, therefore HCO3- can not be directly reabsorbed.   
In tubular fluid  Carbonic
Anhydrase of
apical membrane
,                HCO3- + H+ ←  ——— →  H2CO3 → H2O +CO2
The CO2 diffuses down its concentration difference into tubular cell .
            In tubular cell  Carbonic
anhydrase
              CO2 + H2O ←———→ H2CO3 → H+ + HCO3-
The majority of HCO3- are transported along with Na+ into interstitial fluid and then blood, while the H+ is actively secreted into tubular fluid by Na+-H+ exchange.
Therefore, the manner of HCO3- reabsorption is CO2 .
In the distal tubule and collecting duct, the intercalated cell actively secretes H+, its process is following:
There are much CO2  in the intercalated cells. These CO2 are from both the metabolism of cell and blood.
        In intercalated cell  Carbonic
anhydrase
              CO2 + H2O ←———→ H2CO3 → H+ + HCO3-     
The HCO3- is reabsorbed into interstitial fluid, while the H+ is actively transported by proton pump( H+- ATPase)and H+-K+ ATPase into tubular fluid.
Significance: ①Whenever a H+ is secreted , there must be a HCO3- is generated or reabsorbed. Kidneys maintains the acid-base balance of internal environment by reabsorbing HCO3- and secreting H+. When pH value of internal environment lowers, the activity of carbonic anhydrase increases, secretion of H+ and reabsorption of HCO3- increasing. .② It promotes NH3 secretion. H+ in tubular fluid bind with NH3 to form NH4+. NH4+concentration of tubular fluid decreasing, NH3 secretion increasing.
13. Giving intravenously hyperosmotic glucose solution to a animal (or person), how changes its (or his ) quantity of urine ? why ?
Answer:
Giving intravenously hyperosmotic glucose solution to a animal (or person) leads to the osmotic diuresis. The quantity of urine increases.
Mechanesm: When the hyperosmotic glucose solution is injected intravenously, the blood glucose concentration rises excessively and is higher than the renal glucose threshold. The glucose can not be reabsorbed completely. The excessive glucose remains in the tubular fluid, the osmotic pressure of tubular fluid increases, which retards water reabsorption, the quantity of urine  increasing. For the same reason, the quantity of urine of the most of parients with diabetes increases(hyperuria).
14. When a person is subjected a large amount of sweating and deprivation of drinking water, how change his quantity of urine? Way?
Answer:
His quantity of urine decreases.
Mechnism: The reasons for this are: (1) His plasma crystal osmotic pressure increases. The increased  plasma crystal osmotic pressure stimulates the osmoreceptor in hypothalamus and thus causes posterior pituitary to release ADH. The increased ADH plasma concentration elevates the permeability of distal convoluted tubule and collecting duct to water, thereby increasing water reabsorption, the quantity of urine decreasing and urine concentrates.
(2) His blood volume decrease. The stimulation of the decreased blood volume to the volume receptors located on left atrium and thoracic large vines (such as vena cava, pulmunary vines) decreases; the afferent signals of the vagus nerve decreases; the release of ADH increases reflexly. Finally, the quantity of urine decreases for the same reason.
Conversely, when a person drinks a large amount of water, the opposite sequence of events occurs.
15. When a person drinks a large amount of water, how change his quantity of urine? Way?
  Answer:
  His quantity of urine increases
  Mechanism: (1) After drinking a large amount of water, his plasma is diluted. The stimulation of decreased plasma crystal osmotic pressure to the osmoreceptor in hypothalamus weaken and thus causes the release of ADH to reduce. The decreased ADH plasma concentration reduces the permeability of distal convoluted tubule and collecting duct to water. The reabsorption of water decreases, the quantity of urine increasing.
  (2) After drinking a large amount of water, his blood volume increases. The increased blood volume stimulates the volume receptors located on left atrium and thoracic large vines, the afferent signals of the vagus nerve increasing, the release of ADH reducing reflexly. For the same reason, the quantity of urine increases.   
16. Describe the function and its functional mechanisms of aldosterone on renal tubule and collecting duct
The aldosterone promotes the Na+ reabsorption and K+ secretion in distal convoluted tubule and collecting duct.
Mechanism: Aldosterones enter the principal cells; bind with the receptor in cytoplasm to form a hormone-cytoplasm receptor complex. The complex are transported into the nucleus, then bind with the receptor in the nucleus to form hormone-nucleus receptor complex and thus activates the transcription(转录)process of specific genes to form specific mRNA. The mRNA diffuses into cytoplasm, where it promotes the translation(转化)process to form aldosterone-induced proteins. The latter ① increase the number of the Na+ channels and thus increase the Na+ permeability of the luminal membrane , ② enhance the activity of the Na+ - K+ ATPase pump on the basolateral membrane, ③ increase the synthesis of ATP.
17. Describe the mechanism by which the renal medullary osmolarity gradient formed.
(1). Mechanism forming osmolarity gradient in outer medulla
In outer medulla, the increased osmolarity of interstitial fluid result mainly from the active reabsorption of Na+ and Cl-, while water can not be reabsorbed in the ascending limb thick segment of Henle’s loop. The ascending limb thick segment of Henle’ s loop is impermeable to water, actively reabsorbs Na+ and Cl-, consequently, the NaCl concentration in interstitium is increased, rising osmolarity of interstitial fluid in outer medulla .
(2) . Mechanism forming osmolarity gradient in inner medulla
In inner medulla, the increased concentration of NaCl and urea contribute equally to the increased osmolarity of interstitial fluid .
The process forming the osmolarity gradient of inner medulla is countercurrent multiplication,  is following:
1)The descending limb thin segment of of Henle’s loop is impermeable to urea and NaCl, is highly permeable to water. As tubular fluid flow through this region, water is net reabsorbed, the concentration of NaCl in tubular fluid progressively increases.
2)When this tubular fluid rounds the bend of Henle’s loop and flow into ascending limb thin segment, because the ascending limb thin segment is highly permeable to NaCl and is impermeable to water, NaCl diffuses down concentration difference from tubular fluid into inner medullary peritubular fluid, thereby increasing its osmolarity.
3) Urea circulation increases the urea concentration of inner medulla
The distal convoluted tubule and cortical segment and outer medullary segment of collecting duct is impermeable to urea. In the presence of high concentration ADH, water is reabsorbed from these segments, increasing urea concentration in tubular fluid.
Because the inner medullary collecting duct is highly permeable to urea, then urea diffuses down its concentration difference from tubule into peritubular interstitium and thus increase the urea concentration and osmolarity of peritubular fluid in the inner medulla.
The urea in inner medulla diffuses into ascending limb thin segment, then flow through ascending limb thick segment, distal convoluted tubule, outer medullary segment of collecting duct , and finally back to the inner medullary collecting duct where urea diffuses into interstitial fluid again, this process is called urea circulation .
18. Describe the process of urine dilution and concentration.
(1) Urine dilution
In the ascending limb thick segment of Henle’s loop, Na+, K+, Cl- are avidly reabsorbed, however, this portion of tubule is impermeable to water, therefore the tubular fluid become dilute , its osmolarity is about 100mmol/L. As these dilute fluid flows through the distal tubule and collecting duct, the Na+, K+,Cl- is continuously reabsorbed. In the absence of ADH, these tubule are also impermeable to water. Therefore, the tubular fluid become even more dilute, its osmolarity lowers as low as 50 mOsm/L.
(2) Urine concentration.
The urine concentration occurs in the medullary segment of collecting duct. In presence of high concentration ADH, the distal convoluted tubule and collecting duct is high permeable to water. When tubular fluid flow through the medallary segment of collecting duct, because there is a osmolarity gradient in the interstitial fluid of renal medalla, water moves down osmotic pressure difference from tubular fluid into renal medullary interstitial fluid, up to osmotic equilibrium is achieved, consequently, the quantity of urine decreases and urine osmolarity increases.
离线sdyxs
发帖
712
啄木币
766
鲜花
50
只看该作者 4楼 发表于: 2007-12-15
辛苦了,呵呵
离线平行线
发帖
543
啄木币
1054
鲜花
132
只看该作者 5楼 发表于: 2007-12-24
感谢啊,再有其他章节的吗?
离线dufly
发帖
12
啄木币
119
鲜花
1
只看该作者 6楼 发表于: 2007-12-25
thank you@
离线dufly
发帖
12
啄木币
119
鲜花
1
只看该作者 7楼 发表于: 2007-12-25
为你欢呼!
离线云天之上
发帖
1021
啄木币
1
鲜花
60
只看该作者 8楼 发表于: 2007-12-25
    谢了哈。。。
离线潇湘妃子
发帖
51
啄木币
244
鲜花
18
只看该作者 9楼 发表于: 2008-01-07
非常感谢了
发帖
3031
啄木币
2690
鲜花
242
只看该作者 10楼 发表于: 2008-01-07
如此之多阿
离线红果果
发帖
371
啄木币
578
鲜花
103
只看该作者 11楼 发表于: 2008-01-07
有汉语的么?〉
发帖
2991
啄木币
457
鲜花
323
只看该作者 12楼 发表于: 2008-01-07
  辛苦啦。。
离线冥王转世
发帖
354
啄木币
531
鲜花
20
只看该作者 13楼 发表于: 2008-05-07
呵呵  谢谢
离线狱龙刺
发帖
234
啄木币
433
鲜花
31
只看该作者 14楼 发表于: 2008-08-19
有点乱,第八章貌似出现了三次
快速回复
限100 字节
 
上一个 下一个